Sie sind auf Seite 1von 57

EgyptianPediatrics Yahoo Group

http://health.groups.yahoo.com/group/ EgyptianPediatrics/

Editorial Staff
Editor in Chief: Alistair G.S. Philip, Palo Alto, CA Associate Editor: Josef Neu, Gainesville, FL Assistant Editor, CME: Henry C. Lee, Palo Alto, CA Assistant Editor, Visual Diagnosis, Video Corner: JoDee Anderson, Portland, OR Editorial Board: Dara Brodsky, Boston, MA Robert Castro, Palo Alto, CA Joseph R. Hageman, Evanston, IL Ivan Hand, Great Neck, NY M. Gary Karlowicz, Norfolk, VA Jane McGowan, Philadelphia, PA Steven A. Ringer, Boston, MA Renate Savich, Albuquerque, NM Karen Shattuck, Galveston, TX William Truog, Kansas City, MO Founding Editor: William W. Hay Jr, Denver, CO International Advisory Board: Claudine Amiel-Tison, Paris, France Malcolm Battin, Auckland, New Zealand Matts Blennow, Stockholm, Sweden Jose Diaz Rossello, Montevideo, Uruguay Lex Doyle, Melbourne, Australia Janusz Gadzinowski, Poznan, Poland Gorm Greisen, Copenhagen, Denmark Kazushige Ikeda, Tokyo, Japan Ian Laing, Edinburgh, Scotland Frank Pohlandt, Ulm, Germany Jorge Csar Martinez, Buenos Aires, Argentina Siddarth Ramji, New Delhi, India Francesco Raimondi, Naples, Italy Eric Shinwell, Jerusalem, Israel Bo Sun, Shanghai, China Cleide Trindade, Sao Paolo, Brazil Maximo Vento, Valencia, Spain Andrew Whitelaw, Bristol, United Kingdom David Woods, Cape Town, South Africa Khalid Yunis, Beirut, Lebanon Tsu-Fuy Yeh, Taichung, Taiwan Liaison, Council on International Neonatal Nurses: Carole Kenner, Boston, MA Liaison, National Association for Neonatal Nurses: Susan Reinarz, PhD, DNP, ARNP, NNP-BC, Grand Prairie, TX Managing Editor: Luann Zanzola Editorial Associate: Kathleen Bernard Publisher: American Academy of Pediatrics Associate Executive Director for Education: Robert Perelman Division of Scholarly Journals Director: Michael Held

NeoReviews
Articles

contents
Vol.14 No.2 February 2013 Topics in Neonatal Informatics: Infants and Data in the
Electronic Health Record Era
Eugenia K. Pallotto, Patricia G. Hunt, Francine D. Dykes, David J. Durand, Karna Murthy

e57 e63 e74 e83 e89 e91 e100

Neonatal Cholestasis
Amy G. Feldman, Ronald J. Sokol

Neonatal Thrombocytopenia
Karen S. Fernndez, Pedro de Alarcn

NeoReviews
(ISSN 1526-9906) is owned and controlled by the American Academy of Pediatrics. It is published monthly by the American Academy of Pediatrics, 141 Northwest Point Blvd., Elk Grove Village, IL 60007-1098. Statements and opinions expressed in NeoReviews are those of the authors and not necessarily those of the American Academy of Pediatrics or its Committees. Recommendations included in this publication do not indicate an exclusive course of treatment or serve as a standard of medical care. Subscription price for NeoReviews for 2013: AAP Member $115; AAP National Affiliate Member $86; Nonmember $130; Allied Health/In-training $101; AAP Perinatal Section Member $101. Institutions call for pricing (866-843-2271). AMERICAN ACADEMY OF PEDIATRICS, 2013. All rights reserved. Printed in USA. No part may be duplicated or reproduced without permission of the American Academy of Pediatrics. POSTMASTER: Send address changes to NEOREVIEWS, American Academy of Pediatrics, 141 Northwest Point Blvd., Elk Grove Village, IL 60007-1098. NeoReviews is supported, in part, through an educational grant from Abbott Nutrition, a division of Abbott Laboratories, Inc.

Hemolytic Disease of the Fetus and Newborn


Mary Beth Ross, Pedro de Alarcn

Index of Suspicion in the Nursery: Preterm Infant Born


to Mother With High Fever
Dinushan Kaluarachchi, Tomas Munoz, Suresh Khanna, Yekaterina Sitnitskaya, Benamanahalli Rajegowda

Strip of the Month: February 2013


Maurice L. Druzin, Nancy Peterson

Visual Diagnosis: Term Infant With Respiratory Distress


and Desaturations
Alice Hensley, Bernardo Kracer

NeoReviews Editorial Board Disclosures


The American Academy of Pediatrics (AAP) Policy on Disclosure of Financial Relationships and Resolution of Conflicts of Interest for AAP CME Activities is designed to ensure quality, objective, balanced, and scientifically rigorous AAP CME activities by identifying and resolving all potential conflicts of interest before the confirmation of service of those in a position to influence and/or control CME content. All individuals in a position to influence and/or control the content of AAP CME activities are required to disclose to the AAP and subsequently to learners that the individual either has no relevant financial relationships or any financial relationships with the manufacturer(s) of any commercial product(s) and/or provider(s) of commercial services discussed in CME activities. Commercial interest is defined as any entity producing, marketing, reselling or distributing health-care goods or services consumed by, or used on, patients. Each of the editorial board members, reviewers, question writers, and staff has disclosed, if applicable, that the CME content he/she edits/ writes/reviews may include discussion/reference to generic pharmaceuticals, off-label pharmaceutical use, investigational therapies, brand names, and manufacturers. None of the editors, board members, reviewers, question writers, or staff has any relevant financial relationships to disclose unless noted below. The AAP has taken steps to resolve any potential conflicts of interest.
NeoReviews offers 24 CME articles per year. A maximum of one AMA PRA Category 1 CreditTM is earned after achieving a 60% score on each designated quiz. CME statements: The American Academy of Pediatrics (AAP) is accredited by the Accreditation Council for Continuing Medical Education (ACCME) to provide continuing medical education for physicians. The AAP designates this journal-based CME activity for a maximum of 1.00 AMA PRA Category 1 CreditTM. Physicians should claim only the credit commensurate with the extent of their participation in the activity. This activity is acceptable for a maximum of 1.00 AAP credit. These credits can be applied toward the AAP CME/CPD* Award available to Fellows and Candidate Members of the AAP. The American Academy of Physician Assistants accepts certicates of participation for educational activities certied for AMA PRA Category 1 CreditsTM from organizations accredited by ACCME. Physician assistants may claim a maximum of 1.00 hour of Category 1 credit for completing this program. This program is accredited for 1.00 NAPNAP CE contact hour; pharmacology (Rx) contact hours to be determined per the National Association of Pediatric Nurse Practitioners (NAPNAP) Continuing Education Guidelines.

Disclosures

JoDee Anderson, MD, MEd, FAAP, disclosed that she has a paid consultant relationship with SimHealth. Josef Neu, MD, FAAP, disclosed that he serves as a consultant to Abbott Nutrition, Mead Johnson, Medela, and Fonterra Foods; he receives honorarium from Nestl and Danone; and he has a research grant relationship with Covidien and Gerber.

Answer key appears on page e99.

It has been established that each CME activity will take the learner approximately 1 hour to complete. *Continuing Professional Development How to complete this activity NeoReviews can be accessed and reviewed in print or online at http://neoreviews.aappublications.org. Learners can claim credit monthly online upon completion of each CME article. The deadline for completing this activity is December 31, 2015. Credit will be recorded in the year in which it is submitted. It is estimated that it will take approximately 1 hour to complete each CME article. This activity is not considered to have been completed until the learner documents participation in that activity to the provider via online submission of answers. Course evaluations are online.

Topics in Neonatal Informatics: Infants and Data in the Electronic Health Record Era Eugenia K. Pallotto, Patricia G. Hunt, Francine D. Dykes, David J. Durand and Karna Murthy Neoreviews 2013;14;e57 DOI: 10.1542/neo.14-2-e57

The online version of this article, along with updated information and services, is located on the World Wide Web at: http://neoreviews.aappublications.org/content/14/2/e57

Neoreviews is the official journal of the American Academy of Pediatrics. A monthly publication, it has been published continuously since . Neoreviews is owned, published, and trademarked by the American Academy of Pediatrics, 141 Northwest Point Boulevard, Elk Grove Village, Illinois, 60007. Copyright 2013 by the American Academy of Pediatrics. All rights reserved. Print ISSN: .

topics in neonatal informatics

Infants and Data in the Electronic Health Record Era


Eugenia K. Pallotto, MD, MSCE,* Patricia G. Hunt, MD, MPH,* Francine D. Dykes, MD, David J. Durand, MD, Karna Murthy, MDx

Author Disclosure Drs Pallotto, Hunt, Dykes, Durand, and Murthy have disclosed no nancial relationships relevant to this article. This commentary does not contain a discussion of an unapproved/ investigative use of a commercial product/device.

Abstract
Adoption of electronic health records continues to proceed rapidly. Neonatology offers unique barriers that must be addressed within the continued development of the electronic health record. Reducing these barriers can lead to improvements in the quality and safety of health-care delivery for the neonatal patient.

Objectives

After completing this article, readers should be able to:

1. Gain an understanding of the challenges with implementation of the electronic health record in the neonatal population. 2. Understand the potential opportunities for improving neonatal care with the electronic health record.

Introduction
Electronic health record (EHR) adoption is proceeding rapidly in hospitals in the United States and internationally. The ability to establish a longitudinal picture of a patients health information is viewed as a key benet and requires documenting complete information at each patient encounter. Data from each encounter are then accessible for medical professionals providing care to the patient. For those patients requiring neonatal intensive care due to prematurity or other complex medical issues, it is critically important that an accurate and thorough medical record is started in the NICU. Many of these complex neonatal patients require

Abbreviations
CHND: Childrens Hospital Neonatal Database EHR: electronic health record RDS: respiratory distress syndrome
*Childrens Mercy Hospitals and Clinics, Kansas City, MO. Childrens Healthcare of Atlanta, Atlanta, GA. Childrens Hospital & Research Center Oakland, Oakland, CA. x Ann and Robert H. Lurie Childrens Hospital of Chicago, Chicago, IL.

numerous follow-up visits with pediatric subspecialists throughout their infancy and childhood. The Childrens Hospital Neonatal Database (CHND), implemented in 2010, is a database designed for collection of demographic, interventional, and outcome data on infants admitted to childrens hospitals NICUs. In this data set, as presented at the May 2012 Childrens Hospital Association Forum Series in San Diego, California, more than 50% of patients (entered as of early 2012) had one or two follow-up visits in addition to their primary care providers, and approximately 20% required three or more referrals for outpatient followup. These follow-up needs may also extend into adulthood, particularly as survival with congenital heart disease or other birth anomalies increases. (1)(2)(3) Accurate and complete perinatal and neonatal documentation is imperative to prevent failure in transferring critical information between fetal, pediatric, and adult clinicians.
NeoReviews Vol.14 No.2 February 2013 e57

topics in neonatal informatics

Easy access to historical data allows clinicians to efciently assess trends over time and to avoid duplicate testing or repeat trials of therapy that have previously been documented as ineffective. Longitudinal follow-up is only one reason to promote greater adaptation to EHR systems. These systems support the clinical goals of efcient time use, as well as improvements in the quality and safety of the delivered health-care and clinical decision support. (4) Beyond clinical care, other realms of health-care will benet from an accurate, complete EHR. Patient safety, research, billing, medicolegal purposes, and quality improvement initiatives all rely on accurate, complete health records to accomplish their goals. To this regard, the Centers for Medicare & Medicaid Services has developed meaningful use criteria, incentivizing centers to implement certied EHR technology to achieve their goals for healthcare safety, quality, and efciency. Thus, clinicians and institutions must identify and generate solutions for the inherent challenges that exist within the EHR systems.

Challenges in Data Entry and Quality


Despite the structure within the EHR that allows clinicians to discretely document key pieces of the patients clinical course, failures in consistent documentation of key clinical information such as patient goals have been demonstrated. (5) A review of the impact of the EHR and information quality described the effect of electronic information systems on documentation by health-care professionals. The quality and reliability or precision of documentation vary both between data elements and with different end users. Data accuracy and precision in the EHR increase over time; however, documentation compliance
e58 NeoReviews Vol.14 No.2 February 2013

is shown to be low when decision supports for documentation are deactivated. (6)(7) In addition, variability in diagnosis assignment has been demonstrated both within and across NICUs. Although not an inherent problem due to EHR use, this diagnostic variability has an impact beyond the individual patient. Eichenwald et al (8) demonstrated that inter-NICU variation in the diagnosis of common neonatal problems, such as apnea, had a clinically signicant impact on length of stay and hospital costs. In turn, diagnostic variability reduces the ability to compare populations and outcomes across hospitals. The specicity for coding different neonatal diseases (eg, by using International Classication of Diseases, Ninth Revision codes) is not ideal, leading to potential challenges when interpreting results. Moreover, even when neonatologists can agree on a diagnosis, they rarely agree on the important data elements which led to that diagnosis. (9) With diagnostic variability demonstrated across clinicians, by what criteria should a specic diagnosis be ruled in? For example, did an infant have respiratory distress syndrome (RDS) if the clinician documented RDS or only if the infant met predetermined criteria? Does the data abstractor or does the clinician caring for the patient have the nal say? This issue is not specic to the use of the EHR for documentation; however, the application of the EHR in clinical care and subsequently for data analyses or retrieval either for research or quality improvement efforts highlights this problem. Omitted diagnoses in the medical documentation for infants who did in fact meet predetermined clinical criteria can be particularly problematic. Systematically, these omissions introduce bias in ascertaining cases and, later, in comparing outcomes across centers.

An additional challenge for neonatal data entry in the EHR, specically with very prolonged hospitalizations, is dening the resolution of diagnoses that are known to physiologically resolve. Continuing with our use case of RDS, this problem is typically conned to the rst week after birth for preterm infants. However, it may be frequently carried forward over the hospitalization and even to discharge, thereby reducing the specicity for which true RDS is actually referencing. This nding represents a challenge when extracting data, requiring significant training to determine if RDS was present, if it did resolve, and if not, what diagnosis was actually present (because the biology is not consistent with RDS persisting beyond the rst week after birth). In addition, this trend contributes to over-ascertainment of RDS when a chronic and likely different cause for pulmonary disease becomes the dominant clinical problem (eg, evolving bronchopulmonary dysplasia or pneumonia). Regardless, with analyses, the incomplete or inaccurate resolution of time-limited problems will tend to contribute to bias when EHR data sets are generated and analyzed.

Challenges in Data Extraction


At rst glance, EHR documentation should yield straightforward information regarding key clinical elds or diagnoses that reect an infants hospital course. However, the conicting and/or missing information can lead to challenges when data are retrieved for clinical care, research, quality improvement initiatives, or other reporting mandates. Multiple providers routinely record data in an individual patients EHR and may be documenting the same information, but results may be conicting. Gestational age is one example in which clinical estimates and use of last menstrual period gestational

topics in neonatal informatics

age assignments may vary. This variation increases in the presence of prematurity, perhaps the most important time to have accuracy, because the impact on tracking outcomes and reporting health outcomes is so great in this population of patients. (10) Conicting results lead to questions and confusion when reviewing data retrospectively and thus require clinicians to determine, What is the source of truth? This variation may occur across patients but also within individual patients over time even within an institution. In one review of practices in a NICU, clinical documentation by resident physicians reported discrepancies in 61.7% of notes with respect to weight, vascular lines, or prescribed medications. (11) Although resident physicians are still learning their trade, these discrepancies highlight areas of future work to improve documentation in this setting. Before attempting to obtain benchmarking data or data collection for other purposes, standardization of eld ascertainment and denitions must be completed. Without these steps, comparability is undermined, and actionable interventions become challenging to develop and apply. In one pediatric hospital, signicant differences in adverse drug event rates were identied from different data sources within this institution despite use of EHR tools. (12) Because the focus of the EHR development has primarily been on structured data element development, there have not been similar efforts in assessment of data accuracy or precision, particularly as these systems are upgraded and changed over time. (13) The EHR offers an opportunity to capture and to analyze large amounts of data. However, to date, efforts to do so by using electronic data transfers have been fraught with challenges. De novo EHR systems typically do not

have the capability for data management, clinical decision support, statistical testing, or reporting without enhancements. Inconsistent formats limit electronic data transfers, although efforts to overcome these challenges are underway. One such effort is the Unied Medical Language System, which uses les and software to link health and biomedical vocabularies and standards, thus enabling interoperability between computer systems. (14)

Improving Care With the Electronic Health Record


Despite these challenges, the EHR offers fantastic opportunities to improve care systematically for the neonatal patient, provided that reliable and accurate data entry and extraction are the foundation of these opportunities. Studies suggest that to maximize documentation quality and reliability, focused strategies are needed to support clinicians. Collaborative and educational efforts are needed to identify solutions to minimize and ideally eliminate this variability. These efforts are not only important for quality health-care, but accuracy and appropriate documentation have a role in both medical liability and risk management. (15) The use of decision support within the EHR offers another potential improvement for these issues. Providing clinicians or other health-care stakeholders with the pertinent knowledge can enhance care. (16) Computergenerated recommendations have been shown, in randomized controlled trials, to improve patient care when offered at the time of decisionmaking and within the clinicians workow. (17) Specic studies have demonstrated the benets of clinical decision support in the NICU for improving efciency, particularly as it relates to medication administration

time and efciencies. (18)(19)(20) (21) Reminders to clinicians to rule in, to rule out, to resolve, and to exclude certain diagnoses or even diagnostic categories for various patient populations would seem like the obvious solution to improve care systematically with the EHR. In real time, user prompts can be designed to conrm that the diagnoses assigned meet criteria established a priori and even exclude categories that are unlikely to meet the clinical circumstances (eg, hypospadias in a female patient). With RDS, the assignment of this diagnosis in infants born at 40 weeks gestation is exceptionally unlikely, and support for clinicians to generally avoid this assignment would be helpful. However, decision support requires an investment to plan the content of the tools as well as the costs to design, test, and implement these rules, which, with new evidence, are expected to change over time. Moreover, training each end user is an investment that, to date, would fall primarily on clinical providers and institutions. Presently, the costs and efforts to achieve this level of support are not feasible for the neonatal patient, particularly because these patients comprise a small portion of the population in many hospital systems. One EHR strategy for improving these areas is the development and utilization of NICU-specic templates and order sets. Appropriately designed templates can support clinicians in their efforts to document complete and pertinent information. For example, with the documentation of the insertion of an umbilical artery line, a template can guide clinicians in complete delineation of the events surrounding the procedure (Table 1). Templates can also be developed that will display data that have been recorded in other areas of the EHR within the clinicians
NeoReviews Vol.14 No.2 February 2013 e59

topics in neonatal informatics

notes, thus avoiding transcription errors. Similarly, appropriately designed NICU order sets offer a method for accurate, consistent order entry (Table 2). Although templates and order sets may require extensive time and personnel resources to develop and then maintain (as well as update as practices change), the ultimate goal with their use and implementation is improved efciency, accuracy, and safety. Integration and organization of patients health information to facilitate distribution of the information among the team of providers caring for patients are needed. Handoff tools integrated into the EHR in the NICU improve perception of sign-out accuracy and provider satisfaction. (22) The EHR has the potential to improve the coordination

Table 2.

Example of NICU Insulin Order Set

Infusions for patients 1.9 kg Regular insulin 0.5 unit/mL in 0.45% sodium chloride, 0.1 unit/kg per h, IV Regular insulin 0.5 unit/mL in 0.9% sodium chloride, 0.1 unit/kg per h, IV Infusions for patients 2 kg Regular insulin 1 unit/mL in 0.45% sodium chloride, 0.1 unit/kg per h, IV Regular insulin 1 unit/mL in 0.9% sodium chloride, 0.1 unit/kg per h, IV Bolus dosing Regular insulin 1 unit/mL, 0.1 unit/kg, subcutaneously, once
IVintravenously.

Template: Content for Umbilical Artery Catheter Insertion Procedure Note


Table 1.

of care and decrease fragmentation particularly benecial for those patients (neonates) requiring sophisticated, high-quality medical care. (23) Complex neonatal patients, cared for by neonatologists from their entry into the health-care system, often require multiple specialist evaluations and evaluations in multiple care settings. Using the EHR to coordinate care through information can reduce medical errors, unnecessary testing, and increase the chances that each specialist knows about relevant conditions managed by other specialists. The neonatologist coordinates many of these issues while the neonate is an inpatient. Establishing a solid foundation for documentation

of the conditions begins the steps toward longitudinal continuity of care for our patients. Collaboration and coordination within and across centers are imperative to moving forward on these opportunities. Efforts in improving neonatal knowledge, quality, and benchmarking initially established by the efforts of the Vermont Oxford Network and the Eunice Kennedy Shriver National Institute of Child Health and Human Developments Neonatal Research Network, along with other national and international neonatal collaborations, continue to expand. (24)(25) For example, the CHND began data collection in 2010 and is challenged with the goal

Date and time of procedure: Time out: Performing clinician: Supervising clinician: Informed consent: Indication for placement of umbilical artery catheter: Preparation: Comfort measures provided: Sterile preparation: Premedications administered: Technique: Catheter size inserted: Centimeters catheter advanced into umbilical artery: Estimated blood loss: Conrmation of location: Catheter tip location: Adjustments made to catheter position: Complications:

Figure. Gastroschisis patient care interfacility variation provides data for possible

collaborative quality efforts. Data presented at the May 2012 Childrens Hospital Association Forum Series in San Diego, California. CHND[Childrens Hospital Neonatal Database.

e60 NeoReviews Vol.14 No.2 February 2013

topics in neonatal informatics

of continuing to harmonize data elds across databases and institutions while expanding the ability to benchmark outcomes for uncommon neonatal diagnoses. Variation in the percentage of ventilator days per patient days, across CHND centers, provides an example of variation in care that collaborative efforts could better understand and, through quality improvement, minimize variation when clinically appropriate (Fig). Capturing clinical data for quality improvement efforts is an expensive undertaking but remains essential to continue to improve the quality and the costeffectiveness of medical care within neonatology. Utilization of the EHR to streamline documentation and increase efciencies of data retrieval while maintaining clinically relevant medical records should be feasible. As the data systems that are primarily designed for administrative monitoring are further developed, care must be taken not to impede the clinical process. Efforts at balancing standardization and reporting needs must also ensure that clinicians workow and efciency needs are met as well.

Advancing real-time decision support systems to increase the validity of entered data as well as the clinicians workow efciencies are urgently needed. Attempting to reduce the barriers of varying diagnoses and documentation strategies will foster further clinical care, research aims, and quality improvement initiatives to improve the quality and safety of health-care delivered for the neonatal patient. Multi-institutional efforts are needed to meet these goals. In addition, we must engage national and state policy makers to prioritize neonatology quality measures and health information exchange standards so that the vendors will follow suit. ACKNOWLEDGMENTS. The authors acknowledge the assistance of the executive council members of the Childrens Hospitals Neonatal Consortium: Jacquelyn Evans, MD; Jeanette Asselin, MS, RT; Michael Padula, MD; Kristina Reber, MD; and Billie Lou Short, MD.

3. Carden KA, Boiselle PM, Waltz DA,


Ernst A. Tracheomalacia and tracheobronchomalacia in children and adults: an in-depth review. Chest. 2005;127(3): 9841005 4. Christensen K, Juel K, Herskind AM, Murray JC. Long term follow up study of survival associated with cleft lip and palate at birth. BMJ. 2004;328(7453):1405 5. Collins SA, Bakken S, Vawdrey DK, Coiera E, Currie LM. Agreement between common goals discussed and documented in the ICU. J Am Med Inform Assoc. 2011; 18(1):4550 6. Hyrinen K, Saranto K, Nyknen P. Denition, structure, content, use and impacts of electronic health records: a review of the research literature. Int J Med Inform. 2008;77(5):291304 7. Haberman S, Rotas M, Perlman K, Feldman JG. Variations in compliance with documentation using computerized obstetric records. Obstet Gynecol. 2007;110(1): 141145 8. Eichenwald EC, Zupancic JA, Mao WY, Richardson DK, McCormick MC, Escobar GJ. Variation in diagnosis of apnea in moderately preterm infants predicts length of stay. Pediatrics. 2011;127 (1):e53e58 9. Brown P, Guerlain S, Gordon P, Bauer D. Variations in faculty assessment of NICU owsheet data: implications for electronic data display. Int J Med Inform. 2011;80(7): 529532 10. Mustafa G, David RJ. Comparative accuracy of clinical estimate versus menstrual gestational age in computerized birth certicates. Public Health Rep. 2001;116 (1):1521 11. Carroll AE, Tarczy-Hornoch P, OReilly E, Christakis DA. Resident documentation discrepancies in a neonatal intensive care unit. Pediatrics. 2003;111(5 pt 1):976980 12. Kahn MG, Ranade D. The impact of electronic medical records data sources on an adverse drug event quality measure. J Am Med Inform Assoc. 2010;17(2):185191 13. Thiru K, Hassey A, Sullivan F. Systematic review of scope and quality of electronic patient record data in primary care. BMJ. 2003;326(7398):1070 14. US National Library of Medicine, National Institutes of Health. Unied Medical Language System (UMLS). UMLS quick start guide. Available at: http:// www.nlm.nih.gov/research/umls/quickstart. html. Accessed August 26, 2012 15. Donn SM. Medical liability, risk management, and the quality of health care.
NeoReviews Vol.14 No.2 February 2013 e61

Conclusions
It is imperative, as we move forward in national efforts for neonatal patients, that we engage EHR vendors to support the clinicians abilities to document accurate, valid, and clearly dened data elements in neonatology both across and within institutions. Initiating, from the time of birth, a medical record that comprehensively captures risk factors (pre-exposure and postexposure), therapies, and results for the neonatal patient can set the stage for the continuum of care throughout the individuals life. Pressure for public reporting and transparency make narrowing this variability in data element ascertainment and documentation paramount.

American Board of Pediatrics NeonatalPerinatal Content Specications


Know the issues in the organization of perinatal care (eg, regionalization, transport quality control, practice guidelines).

References
1. Incentive Programs EHR. The Ofcial
Web Site for the Medicare and Medicaid Electronic Health Records (EHR) Incentive Programs. Available at: https://www.cms. gov/Regulations-and-Guidance/Legislation/ EHRIncentivePrograms/index.html. Accessed August 26, 2012 2. Warnes CA, Liberthson R, Danielson GK, et al. Task force 1: the changing prole of congenital heart disease in adult life. J Am Coll Cardiol. 2001;37(5):11701175

topics in neonatal informatics

Semin Fetal Neonatal Med. 2005;10(1): 39 16. Osheroff JA, Teich JM, Middleton B, Steen EB, Wright A, Detmer DE. A roadmap for national action on clinical decision support. J Am Med Inform Assoc. 2007;14(2):141145 17. Kawamoto K, Houlihan CA, Balas EA, Lobach DF. Improving clinical practice using clinical decision support systems: a systematic review of trials to identify features critical to success. BMJ. 2005;330(7494):765768 18. Taylor JA, Loan LA, Kamara J, Blackburn S, Whitney D. Medication administration variances before and after implementation of computerized physician order entry in a neonatal intensive care unit. Pediatrics. 2008;121(1): 123128 19. Cordero L, Kuehn L, Kumar RR, Mekhjian HS. Impact of computerized

physician order entry on clinical practice in a newborn intensive care unit. J Perinatol. 2004;24(2):8893 20. Chapman AK, Lehmann CU, Donohue PK, Aucott SW. Implementation of computerized provider order entry in a neonatal intensive care unit: Impact on admission workow. Int J Med Inform. 2012;81(5):291295 21. Maat B, Rademaker CM, Oostveen MI, Krediet TG, Egberts TC, Bollen CW. The effect of a computerized prescribing and calculating system on hypo- and hyperglycemias and on prescribing time efciency in neonatal intensive care patients [published online April 25, 2012]. JPEN J Parenter Enteral Nutr. Available at http://pen. sagepub.com/content/early/2012/04/23/ 0148607112444608. Accessed December 3, 2012

22. Palma JP, Sharek PJ, Longhurst CA. Impact of electronic medical record integration of a handoff tool on sign-out in a newborn intensive care unit. J Perinatol. 2011;31(5):311317 23. Health IT.gov. Benets of EHRs. Improved care coordination. Available at: http:// www.healthit.gov/providers-professionals/ improved-care-coordination. Accessed August 26, 2012 24. Horbar JD, Plsek PE, Leahy K, Ford P. The Vermont Oxford Network: improving quality and safety through multidisciplinary collaboration. NeoReviews. 2004;5(2): e42e49 25. NICHD Neonatal Research Network. Background and overview. Available at: https://neonatal.rti.org/about/network.cfm. Accessed August 26, 2012

e62 NeoReviews Vol.14 No.2 February 2013

Topics in Neonatal Informatics: Infants and Data in the Electronic Health Record Era Eugenia K. Pallotto, Patricia G. Hunt, Francine D. Dykes, David J. Durand and Karna Murthy Neoreviews 2013;14;e57 DOI: 10.1542/neo.14-2-e57

Updated Information & Services References

including high resolution figures, can be found at: http://neoreviews.aappublications.org/content/14/2/e57 This article cites 20 articles, 10 of which you can access for free at: http://neoreviews.aappublications.org/content/14/2/e57#BIBL This article, along with others on similar topics, appears in the following collection(s): Fetus and Newborn Infant http://neoreviews.aappublications.org/cgi/collection/fetus_newb orn_infant Information about reproducing this article in parts (figures, tables) or in its entirety can be found online at: /site/misc/Permissions.xhtml Information about ordering reprints can be found online: /site/misc/reprints.xhtml

Subspecialty Collections

Permissions & Licensing

Reprints

Neonatal Cholestasis Amy G. Feldman and Ronald J. Sokol Neoreviews 2013;14;e63 DOI: 10.1542/neo.14-2-e63

The online version of this article, along with updated information and services, is located on the World Wide Web at: http://neoreviews.aappublications.org/content/14/2/e63

Neoreviews is the official journal of the American Academy of Pediatrics. A monthly publication, it has been published continuously since . Neoreviews is owned, published, and trademarked by the American Academy of Pediatrics, 141 Northwest Point Boulevard, Elk Grove Village, Illinois, 60007. Copyright 2013 by the American Academy of Pediatrics. All rights reserved. Print ISSN: .

Article

gastrointestinal disorders

Neonatal Cholestasis
Amy G. Feldman, MD,* Ronald J. Sokol, MD

Educational Gaps
1. Early diagnosis of neonatal cholestasis is potentially life-saving; however, delayed diagnosis remains a problem. 2. There are several key steps in evaluating the patient who has cholestasis, and following these steps in a timely manner is crucial to identifying the underlying etiology. 3. Biliary atresia (BA) is the most common cause of cholestasis, and although an effective BA screening program was created in Taiwan and is being initiated in many countries around the world, the programs success is not assured in the United States because there is no standard 1-month infant health provider visit, in spite of the public health benet.

Author Disclosure Drs Feldman and Sokol have disclosed no nancial relationships relevant to this article. This commentary does contain a discussion of an unapproved/ investigative use of a commercial product/ device.

Abstract
Cholestatic jaundice is a common presenting feature of neonatal hepatobiliary and metabolic dysfunction. Any infant who remains jaundiced beyond age 2 to 3 weeks should have the serum bilirubin level fractionated into a conjugated (direct) and unconjugated (indirect) portion. Conjugated hyperbilirubinemia is never physiologic or normal. The differential diagnosis of cholestasis is extensive, and a step-wise approach based on the initial history and physical examination is useful to rapidly identify the underlying etiology. Early recognition of neonatal cholestasis is essential to ensure timely treatment and optimal prognosis. Even when specic treatment is not available, infants who have cholestasis benet from early medical management and optimization of nutrition. Future studies are necessary to determine the most reliable and costeffective method of universal screening for neonatal cholestasis.

Objectives

After completing this article, readers should be able to:

1. Understand when a jaundiced infant needs evaluation for cholestatic liver disease. 2. List the differential diagnosis for cholestatic liver disease of the neonate and identify those causes that are amenable to immediate medical or surgical intervention. Abbreviations 3. Describe the step-wise approach to evaluation of a cholestatic infant. A1AT: a1-antitrypsin BA: biliary atresia 4. Understand the importance of early screening for GGT: g-glutamyl transpeptidase cholestatic liver disease and be aware of new research HPE: hepatic portoenterostomy suggesting the importance of early laboratory values in INH: idiopathic neonatal hepatitis identifying cholestatic infants. PFIC: progressive familial intrahepatic cholestasis
PN: parenteral nutrition PNAC: parenteral nutritionassociated cholestasis SBS: short bowel syndrome

Introduction
Jaundice, a yellow discoloration of the skin, sclera, mucous membranes, and bodily uids, is a common clinical nding

*Fellow in Pediatric Gastroenterology, Hepatology and Nutrition, Department of Pediatrics, University of Colorado School of Medicine, and Digestive Health Institute, Childrens Hospital Colorado, CO. Professor and Vice Chair of Pediatrics, Chief of Section of Pediatric Gastroenterology, Hepatology and Nutrition, Department of Pediatrics, and Director of Colorado Clinical and Translational Sciences Institute, University of Colorado Denver, and Digestive Health Institute, Childrens Hospital Colorado, CO.

NeoReviews Vol.14 No.2 February 2013 e63

gastrointestinal disorders

cholestasis

in the rst 2 weeks after birth, occurring in 2.4% to 15% of newborns. (1) Most often, jaundice is of the indirect/ unconjugated bilirubin variety and resolves spontaneously without intervention. However, persistent jaundice is abnormal and can be the presenting sign of serious hepatobiliary and metabolic dysfunction. When jaundice persists beyond age 2 weeks, cholestasis or conjugated hyperbilirubinemia must be considered in the differential diagnosis. Cholestasis represents an impairment in bile ow and may be caused by either an intrahepatic or extrahepatic disorder. To differentiate cholestasis from benign causes of jaundice, the serum bilirubin must be fractionated into conjugated (or direct) and unconjugated (or indirect) fractions. Conjugated hyperbilirubinemia is generally dened as a conjugated or direct bilirubin level greater than 1 mg/dL when the total bilirubin is less than 5 mg/dL or more than 20% of the total bilirubin if the total bilirubin is greater than 5 mg/dL. Conjugated hyperbilirubinemia is never physiologic or normal. Unconjugated hyperbilirubinemia, conversely, is a common nding and can result from physiologic jaundice, breastfeeding and human milkassociated jaundice, red blood cell hemolysis, hypothyroidism, Gilbert syndrome, or Crigler-Najjar syndrome. Clues to the diagnosis of cholestasis include hepatomegaly, diarrhea and poor weight gain, hypopigmented or acholic stools, and dark urine that may stain the diaper. Any infant who remains jaundiced beyond age 2 to 3 weeks needs to be evaluated to rst exclude neonatal cholestasis and, if present, to rapidly identify those causes of cholestasis that are amenable to medical or surgical treatment. Even when specic treatment is not available or curative, infants who have cholestasis benet from early medical management and optimization of nutrition to prevent complications. Despite data showing that early diagnosis of cholestasis and its etiologies is potentially life-saving, (2) delayed diagnosis remains a problem. (3) Early hospital discharge of newborns, inadequate followup of persisting jaundice, false reassurance by the appearance of pigmented stool, uctuating serum bilirubin levels, and misdiagnosis of human milkassociated jaundice are all cited as reasons for late referral for evaluation of cholestasis. (3)(4)(5)

time in life and include infections, anatomic abnormalities of the biliary system, endocrinopathies, genetic disorders, metabolic abnormalities, toxin and drug exposures, vascular abnormalities, neoplastic processes, and other miscellaneous causes (Table 1). (7) Of the many conditions that cause neonatal cholestasis, the most commonly identiable are biliary atresia (BA) (25%35%), genetic disorders (25%), metabolic diseases (20%), and a1-antitrypsin (A1AT) deciency (10%). (8) In older series, idiopathic neonatal hepatitis (INH) was the most common cause of neonatal cholestasis, with a reported incidence of 1 in 4,800 to 1 in 9,000 live births. (9) However, with the discovery of specic etiologies that share the phenotype of INH in addition to more advanced diagnostic methods, the incidence of INH has decreased substantially. In infants born prematurely and in those who have short bowel syndrome (SBS) or intestinal failure, parenteral nutritionassociated cholestasis (PNAC) commonly develops in those receiving parenteral nutrition (PN) for more than 2 to 4 weeks.

Clinical Features
The typical ndings in an infant who has cholestasis are protracted jaundice, scleral icterus, acholic stools, dark yellow urine, and hepatomegaly. It should be noted that there may be a perception of decreasing jaundice over the rst weeks after birth as the indirect bilirubin component (from human milkassociated jaundice) decreases, causing false reassurance that the jaundice is resolving and need not be evaluated further. Acholic stools in an infant should always prompt further evaluation. Some infants may have coagulopathy secondary to vitamin K malabsorption and deciency and present with bleeding or bruising. Coagulopathy may also be caused by liver failure, indicating either severe metabolic derangement of the liver (as in respiratory chain deciency disorders) or cirrhosis and end-stage liver disease (as in neonatal hemochromatosis). Splenomegaly can be observed in infants who have cirrhosis and portal hypertension, storage diseases, and hemolytic disorders. Neurologic abnormalities including irritability, lethargy, poor feeding, hypotonia, or seizures can indicate sepsis, intracranial hemorrhage, metabolic (including Zellweger syndrome) and mitochondrial disorders, or severe liver dysfunction resulting in hyperammonemia and encephalopathy. Low birth weight, thrombocytopenia, petechiae and purpura, and chorioretinitis are often associated with congenital infection. Facial dysmorphism may suggest a chromosomal abnormality or Alagille syndrome. A palpable mass in the right upper quadrant may indicate a choledochal

Etiology
Cholestatic jaundice affects approximately 1 in every 2,500 infants and has a multitude of causes. (6) The number of unique disorders presenting with cholestasis in the neonatal period may be greater than at any other
e64 NeoReviews Vol.14 No.2 February 2013

gastrointestinal disorders

cholestasis

Table 1.

Differential Diagnosis of Neonatal Cholestasis


Genetic and Metabolic A1AT deciency

Infectious Viral (adenovirus; cytomegalovirus; coxsackievirus; Epstein-Barr; echovirus; enterovirus; hepatitis A, B, or C; herpes simplex; human immunodeciency virus; parvovirus; reovirus; rubella) Bacterial (urinary tract infection, sepsis, listeriosis, tuberculosis) Spirochete (syphilis, leptospirosis) Parasites (toxoplasmosis, malaria, toxocariasis) Histoplasmosis

Alagille syndrome Aagenaes syndrome Arthrogryposis, renal dysfunction, and cholestasis syndrome Bile acid synthetic defects Cholestasis of North American Indians Cholesterol synthesis defects Citrin deciency Cystic brosis Dubin-Johnson syndrome Fatty acid oxidation defects (SCAD, LCAD) Galactosemia Glycogen storage disease type 4 GRACILE syndrome Hereditary fructose intolerance Indian childhood cirrhosis Mitochondrial respiratory chain disorders Neonatal iron storage disease Niemann-Pick disease type C Peroxisomal disorders (including Zellweger syndrome) PFIC 1, 2, and 3 (FIC1, BSEP, or MDR3 deciency) Rotor syndrome Lipid storage diseases (eg, Wolman, Gaucher, Farber) Trisomy 13, 18, or 21; Turner syndrome Tyrosinemia Urea cycle defects, arginase deciency Toxins Drugs (ceftriaxone, chloral hydrate, erythromycin, ethanol, isoniazid, methotrexate, rifampin, sulfa-containing products, tetracycline) Total parenteral nutritionassociated cholestasis Herbal products Other Cardiovascular abnormalities Ischemia-reperfusion injury Perinatal asphyxia Extracorporeal membrane oxygenation Budd-Chiari syndrome Veno-occlusive disease Graft-versus-host disease Hemophagocytic lymphohistiocytosis Idiopathic neonatal hepatitis Neonatal lupus erythematosus Malignancy (neonatal leukemia)

Endocrine Hypothyroidism Hypopituitarism (septo-optic dysplasia) McCune-Albright syndrome Anatomic obstruction Biliary atresia Caroli disease Choledochal cyst or other congenital bile duct anomaly Congenital hepatic brosis Gallstones or biliary sludge Inspissated bile syndrome Neonatal sclerosing cholangitis Nonsyndromic bile duct paucity Spontaneous perforation of the bile duct Tumor/mass

A1ATalpha1-antitrypsin, LCADlong chain acyl-CoA dehydrogenase, PFICprogressive familial intrahepatic cholestasis, SCADshort-chain acyl-CoA dehydrogenase, TPGSD-a-tocopheryl polyethylene glycol 1,000 succinate.

NeoReviews Vol.14 No.2 February 2013 e65

gastrointestinal disorders

cholestasis

cyst. A cardiac murmur increases the likelihood of Alagille syndrome or BA. Although 20% of BA patients will have other extrahepatic congenital malformations (including cardiac anomalies, situs inversus, intestinal malrotation, midline liver, and polysplenia or asplenia), the majority of patients who have BA are well appearing during the rst month after birth, and there is no single historical or physical examination nding that uniquely suggests BA.

Evaluation of Neonatal Cholestasis


Evaluation of a jaundiced infant should begin with fractionation of serum bilirubin into total and direct (or conjugated) bilirubin. Infants who have cholestasis will generally have a direct (or conjugated) bilirubin greater than 2.0 mg/dL, which will be more than 20% of the total bilirubin concentration. Recent data suggest that in the rst 4 days after birth, the cutoff for elevated direct bilirubin may be greater than 0.8 mg/dL and more than 8% to 10% of the total bilirubin. (10) Another recent study suggested that in the rst 14 days after birth, the cutoff for elevated conjugated bilirubin may be greater than 0.5 mg/dL, and for direct bilirubin greater than 2 mg/dL. (11) Clearly, further careful study is needed to determine the normal distribution of direct and conjugated bilirubin levels and their percentage of total bilirubin, and to establish abnormal cutoffs based on day of age. If cholestasis is present, further evaluation should be completed with a sense of urgency because patients who have BA have a better outcome if they undergo a Kasai hepatic portoenterostomy (HPE) before age 30 to 45 days, and other conditions (eg, hypothyroidism) require prompt treatment. Levels of liver enzymes, including alanine aminotransferase, aspartate aminotransferase, and alkaline phosphatase, are usually elevated in a cholestatic infant but are poor predictors of etiology. g-Glutamyl transpeptidase (GGT) is generally elevated during cholestasis (particularly in extrahepatic obstructive lesions and those involving intrahepatic bile ducts); however, a low or normal GGT out of proportion to the degree of cholestasis suggests the presence of progressive familial intrahepatic cholestasis (PFIC) type 1, PFIC type 2, an inborn error of bile acid synthesis or metabolism, or panhypopituitarism. GGT may be normal or elevated in PNAC. Baseline albumin, glucose, and prothrombin time/international normalized ratios are useful in assessing the degree of liver synthetic dysfunction. Severe coagulopathy that is unresponsive to parenteral vitamin K suggests synthetic liver failure, metabolic disease, or sepsis. A low serum albumin
e66 NeoReviews Vol.14 No.2 February 2013

level may indicate liver synthetic failure, undernutrition, or protein loss from the kidney or intestine. Depending on the clinical scenario, bacterial cultures from blood and urine may be indicated. The search for congenital viral infection may include a combination of cultures and serologies; immunoglobulin Gbased serologies indicate transplacental transport of maternal immunoglobulin G rather than neonatal infection. The newborn screen can be helpful in identifying galactosemia and hypothyroidism, two treatable causes of cholestasis. An elevated immunoreactive trypsinogen on the newborn screen raises suspicion for cystic brosis and should be followed up with genetic testing and/or a sweat test to determine if the infant has cystic brosis. A low serum A1AT level and an abnormal protease inhibitor phenotype (PIZZ and PISZ) are used to identify A1AT deciency. Other tests that are commonly used to establish a specic diagnosis include urinary-reducing substances or red blood cell galactose-1-phosphate uridyl transferase drawn before any blood transfusions (for galactosemia), urine succinylacetone (for hereditary tyrosinemia), sweat test (for cystic brosis), thyroid-stimulating hormone and thyroxine (for hypothyroidism), total serum bile acid level and urine bile acid prole (for disorders of bile acid synthesis), serum amino acids and urine organic and amino acids (for citrin deciency, fatty acid oxidation defects, and other metabolic diseases), very long chain fatty acid levels (for peroxisomal disorders), and other infectious agent serologies as indicated. Genetic testing for Alagille syndrome, cystic brosis, A1AT deciency, three distinct forms of PFIC, and peroxisomal defects are commercially available. In the near future, next-generation DNA sequencing will allow for multiple genetic tests on small amounts of blood at a relatively low cost. (12) An abdominal ultrasound examination should be obtained as part of the early evaluation of a cholestatic infant to assess liver structure, size, and composition; to evaluate for the presence of ascites; and to identify ndings of an extrahepatic obstructive lesion (choledochal cyst, mass, gallstone, and sludge). Ultrasound ndings suggestive of BA include a triangular cord sign (cone-shaped brotic mass cranial to the bifurcation of the portal vein) or absence of the gallbladder; however, these ndings cannot be reliably used to diagnose BA as they are neither highly sensitive nor specic. (13)(14) Ultrasound can also detect polysplenia or asplenia, interrupted inferior vena cava, preduodenal portal vein, and situs inversus; all of these conditions would strongly suggest BA splenic malformation syndrome and other laterality defects. Common bile duct dilation is not seen in BA and suggests a distal obstruction or a forme fruste choledochal cyst.

gastrointestinal disorders

cholestasis

If a cardiac murmur is appreciated on physical examination, an echocardiogram should be obtained to assess for cardiac anomalies. Up to 24% of patients who have Alagille syndrome and a subset of BA patients will have structural heart disease. A chest radiograph may reveal cardiomegaly or buttery vertebrae in patients who have Alagille syndrome. A careful slit-lamp examination may reveal posterior embryotoxon or other anterior chamber abnormalities in an infant who has Alagille syndrome or chorioretinitis in an infant who has a congenital infection. Hepatobiliary scintigraphy with a technetium-labeled iminodiacetic acid analogue can sometimes be of assistance in distinguishing obstructive from nonobstructive causes of cholestasis. In a healthy infant, injected radioisotype is taken up by the hepatocytes, secreted into the biliary system, and then excreted into the small intestine within 24 hours. Slow uptake of the injected radioisotope or nonvisualization of the liver with persistence of the cardiac pool suggests hepatocellular dysfunction, whereas nonvisualization of the radioisotope in the small intestine at 4 to 24 hours suggests either bile duct obstruction or the severe inability of the hepatocyte to secrete. The sensitivity of scintigraphy for BA is relatively high (83%100%); however, its specicity is low (33%80%), (15)(16) limiting its use to differentiate BA from other nonsurgical conditions. Pretreatment with phenobarbital may increase test sensitivity. Many centers do not routinely use this test in the evaluation of cholestatic infants because it may delay the diagnostic evaluation without providing denitive diagnostic information. At this time, endoscopic retrograde cholangiopancreatography and magnetic resonance cholangiopancreatography are of limited usefulness for the evaluation of neonatal cholestasis. Percutaneous liver biopsy remains an important diagnostic tool in evaluating neonatal cholestasis and can be performed safely in even the smallest infants. In several single-center studies, a diagnosis of BA was correctly suggested by liver biopsy histologic ndings in 90% to 95% of cases. (17) A more recent study suggests a somewhat lower predictive value of liver biopsy ndings when examined in a multicenter research network. (18) Characteristic histologic ndings of BA include bile plugs in the portal tract bile duct, bile ductular proliferation, and portal tract edema and brosis. Results of a liver biopsy can be helpful in establishing other causes of neonatal cholestasis, including A1AT deciency (periodic acid Schiff-positive, diastase-resistant intrahepatocytic globules), Alagille syndrome (bile duct paucity), neonatal sclerosing cholangitis (necroinammatory duct lesions), viral infection (cytomegalovirus or herpes simplex virus inclusions), metabolic liver diseases (steatosis and pseudoacinar formation of hepatocytes),

PFIC and storage diseases (electron microscopy ndings), and INH (multinuclear giant cells, extramedullary hematopoiesis, and hepatocellular cholestasis). Liver histologic ndings in PNAC may resemble all the features of BA and are not useful in differentiating between the two conditions. Repeat liver biopsies may occasionally be needed if the diagnosis is unclear; several of these diseases are dynamic and may not be diagnosable by using results of liver biopsy if performed early in the disease course. In cases in which BA, choledochal cyst, or biliary tract stone disease is suspected, the infant should undergo intraoperative cholangiography through a mini-laparotomy to delineate the biliary anatomy and localize the area of obstruction. The surgeon should be prepared and capable of performing an HPE for BA or choledochal cyst corrective surgery during the same surgical session if these lesions are found on cholangiography. The decision to pursue cholangiography in infants who have SBS with suspected PNAC but who develop acholic stools may be difcult and requires careful consideration of the surgical options if BA is found.

Specic Disorders Resulting in Neonatal Cholestasis


Biliary Atresia
BA occurs in 1 in 6,000 to 18,000 live births and is an idiopathic brosing cholangiopathy of unknown etiology that leads to complete obstruction of the extrahepatic bile duct during the rst few months after birth, progressive biliary cirrhosis, and eventual death if left untreated. It is more common in Asians and African Americans, with a slight female predominance. BA is the leading cause of neonatal cholestasis and the most common reason for pediatric liver transplantation, accounting for 40% to 50% of children who undergo transplantation. The majority of children who have BA appear to be healthy thriving infants who develop or have persisting jaundice and acholic stools at approximately age 3 to 6 weeks. Up to 20% of infants who have BA have congenital malformations, including the BA splenic malformation syndrome (w8%) or other isolated major congenital malformations, the so-called fetal/embryonic form. These infants may appear jaundiced at birth and remain so. The remaining 80% of infants who have BA have isolated atresia without other congenital malformations and are labeled as having the perinatal or so-called acquired form. At the time of diagnosis, an HPE procedure is performed during which a Roux-en-Y loop of intestine is anastomosed to a carefully dissected hilum of the liver to create a conduit for biliary drainage. The rate of success
NeoReviews Vol.14 No.2 February 2013 e67

gastrointestinal disorders

cholestasis

in re-establishing bile ow is dependent on the age of the infant when the HPE is performed. There is up to an 80% success rate if the surgery takes place at less than age 30 to 45 days; however, fewer than 20% of patients who undergo HPE at older than 90 days achieve bile drainage. (2)(19) (20) If jaundice successfully clears after HPE, the 10-year transplant-free survival rate ranges from 75% to 90%; conversely, if jaundice (serum total bilirubin higher than 1.52.0 mg/dL) persists after HPE, the 3-year transplantfree survival rate is 20%. Eventually, the vast majority of patients who have BA have progressive disease, with at least 80% requiring liver transplantation by age 20 years. (21) Of those who survive into the third decade after birth, almost all have portal hypertension or other complications of cirrhosis.

vascular abnormalities (including intracranial lesions in up to 12% of patients), and short stature. (9)(22) The outcome of Alagille syndrome is largely dependent on the individuals particular clinical manifestations, especially the severity of the cardiac and renal lesions. For those presenting with cholestatic liver disease in infancy, 20% to 50% will require liver transplantation or succumb to cardiac or renal disease by age 20 years. (24)

Parenteral NutritionAssociated Cholestasis


Overall, 18% to 67% of infants who receive prolonged courses of PN (longer than 14 days) develop liver injury and cholestasis. (25) The incidence of PNAC is correlated inversely with birthweight and directly with duration of PN therapy. (26) In a study of more than 1,300 infants, the incidence of PNAC increased from 14% in infants who received PN for 14 to 28 days to 86% in those infants who received PN for more than 100 days. Infants who have sepsis, bacterial overgrowth of the small intestine, and intestinal failure (secondary to necrotizing enterocolitis, gastroschisis, or intestinal atresia) are at increased risk for developing PNAC. (26)(27) The presence of cholestasis is the leading predictor of mortality in infants who have short bowel syndrome. (28) The pathogenesis of PNAC is thought to be multifactorial. The soybean-based lipid emulsion component of PN has been implicated as a potential causative factor in PNAC. However, the lipid emulsion component of PN cannot be completely removed because it provides an energy-dense source of calories and essential fatty acids. There is evidence that restriction of the intravenous fat emulsion to 1 g/kg two to three times per week can reduce total bilirubin without causing growth failure or severe essential fatty acid deciency. (29) Therapeutic lipid restriction (to 11.5 g/kg per day) is currently recommended for infants who have developed PNAC. (30) Omegaven (Fresenius, Homburg, Germany), an investigational product in the United States, is a sh oilbased lipid emulsion composed of omega-3 fatty acids instead of omega-6 fatty acids, and is devoid of plant sterols. It has been used as a substitute for the standard soybean-based lipid emulsions, although only at doses of 1.5 g/kg per day. Several case series have reported that Omegaven seems to be safe and effective in reversing PNAC compared with historical controls receiving soy lipidbased lipid emulsions. A prospective clinical trial comparing Omegaven with a standard soybean oil lipid emulsion is underway. (31) Whether Omegaven will reverse the brotic component of PNAC and provide longterm benet is not known. (32) At this time, Omegaven

A1AT deciency is an autosomal recessive disorder, most common in those of Northern European descent and extremely unusual in Asians. It is the most common inherited cause of neonatal cholestasis, affecting approximately 1 in 2,000 live births. Affected individuals have a misfolded A1AT protein that fails to be secreted normally by the hepatocyte, leading to decreased A1AT activity in the blood and lungs and excess retention in hepatocytes. The circulating deciency of A1AT leads to a failure to neutralize neutrophil elastase in the lungs and premature emphysema in young adults. Forty percent to 50% of infants who have the PIZZ phenotype may have asymptomatic abnormal liver biochemical test results in the rst year after birth, and 10% to 15% will develop neonatal cholestasis. However, less than 25% of those presenting with cholestasis will progress to end-stage liver disease during childhood. (22) Eight percent to 15% of patients will develop clinically signicant liver disease during their lifetime. There is no specic treatment for A1AT deciency. Children who develop cirrhosis and liver failure may require liver transplantation.

a1-Antitrypsin Deciency

Alagille Syndrome
Alagille syndrome is an autosomal dominant multisystem disorder characterized by a paucity of intralobular bile ducts and occurring in approximately 1 in 70,000 live births. Almost all patients have a mutation in the JAGGED 1 gene that encodes a ligand in the Notch signaling pathway. Patients who have Alagille syndrome have a combination of neonatal cholestasis and bile duct paucity, congenital heart disease (with peripheral pulmonary artery stenosis being the most common lesion), dysmorphic facies (triangular face, broad forehead and deep-set eyes, small pointed chin, and bulbous nose), buttery vertebrae, ocular posterior embryotoxon, renal anomalies,
e68 NeoReviews Vol.14 No.2 February 2013

gastrointestinal disorders

cholestasis

is restricted to only compassionate use in the United States. Recently, a combination of soybean, mediumchain triglycerides, olive oil, and sh oil lipid emulsions (Fresenius) has been tested in infants who have PNAC. It has shown promising effects on decreasing bilirubin without causing essential fatty acid deciency; however, further investigation of the effects of this combination in infants who have intestinal failure is needed. (30)(33) Infants receiving PN should be started on enteral feedings as early as possible to stimulate bile ow, gallbladder contraction, and intestinal motility. Even trophic feeds have been shown to be benecial in reducing the incidence and severity of PNAC. For patients who have PNAC who continue on PN, the manganese and copper in PN solutions should be reduced and plasma levels monitored because these metals can accumulate to toxic levels in the cholestatic liver. Fat-soluble vitamin levels should be closely monitored and total PN solutions adjusted accordingly. Ursodeoxycholic acid is theoretically of benet by stimulating bile ow; however, there is no evidence of its efcacy in PNAC. High-dose oral erythromycin resulted in lower serum direct bilirubin in one large trial in preterm infants receiving total PN. (34)

progressive cholestasis of hepatocellular origin. In PFIC type 1 (FIC1 [an aminophospholipid ippase]; Byler disease) and PFIC type 2 (BSEP [the ATP-dependent bile acid transporter] deciency), patients typically have low or normal GGT levels and low cholesterol levels and develop early cholestasis. Patients who have PFIC 1 may also have severe diarrhea, pancreatitis, and hearing loss. Severe pruritus develops before age 1 year. Many of these patients respond to partial biliary diversion or ileal exclusion surgery. (35)(36) Unresponsive patients may require liver transplantation in the rst decade after birth. Patients who have PFIC type 3 (MDR3 [canalicular phospholipid transporter] deciency) have cholestasis with elevated GGT and low biliary phospholipids, bile duct inammation, and proliferation on liver biopsy, and they develop biliary cirrhosis rather quickly during childhood. Pruritus is less severe than in the other forms of PFIC and is often responsive to ursodeoxycholic acid.

Treatment of Neonatal Cholestasis


It is crucial to rapidly identify infants who have medically treatable forms of cholestasis as well as those causes amenable to surgical intervention (Table 2). The timing of HPE in patients who have BA is critical. In a recent French study of 695 patients who have BA, survival with native liver was best in children who underwent the HPE procedure in the rst 30 days after birth. (2)

Galactosemia

Galactosemia is an autosomal recessive disorder that occurs in 1 in 50,000 live births. A deciency of the enzyme galactose-1-phosphate uridyl transferase results in defective metabolism of galactose. Newborn screening for Table 2. galactosemia is performed in most countries, thus identifying the majority of infants before they become Cause of Cholestasis symptomatic. However, infants who Infection (bacterial or viral) have galactosemia may present with Galactosemia failure to thrive, vomiting, diarrhea, Tyrosinemia cataracts, Escherichia coli septicemia, jaundice and cholestasis, hepatomegaly, ascites, or hypoglycemia. Hereditary fructose intolerance Hypothyroidism Treatment of galactosemia involves Cystic brosis dietary avoidance of all foods that Hypopituitarism contain galactose and lactose.
Bile acid synthetic defect

Causes of Cholestasis That Require Specic Medical or Surgical Intervention


Intervention Antibiotic or antiviral agents Galactose-free diet Low tyrosine/phenylalanine diet, 2-(2-nitro-4-triuoromethylbenzol)1,3-cyclohexanedione Fructose- and sucrose-free diet Thyroid hormone replacement Pancreatic enzymes, ursodeoxycholic acid Thyroid, growth hormone, cortisol replacement Ursodeoxycholic or cholic acid supplementation Hepatoportoenterostomy (Kasai procedure) Choledochoenterostomy Surgical drainage Biliary tract irrigation

Progressive Familial Intrahepatic Cholestasis


PFIC is a group of at least three autosomal recessive hereditary disorders in which mutations in one of the genes involved in canalicular bile formation results in

Biliary atresia Choledochal cyst Spontaneous perforation of the common bile duct Inspissated bile in the common bile duct

NeoReviews Vol.14 No.2 February 2013 e69

gastrointestinal disorders

cholestasis

Nutritional therapy is of utmost importance in cholestatic infants. Growth failure is common secondary to impaired absorption of fats, impaired metabolism of proteins and carbohydrates, and increased metabolic demand. Reduced delivery of bile acids to the small intestine leads to decreased mixed micelle formation and subsequent fat and fat-soluble vitamin malabsorption. Caloric intake should be approximately 125% of the recommended dietary allowance based on ideal body weight. Adequate protein intake of 2 to 3 g/kg per day should be delivered. Cholestatic infants should receive a formula containing medium-chain triglycerides, such as Pregestimil (Mead Johnson & Company, Evansville, IN) or Alimentum (Abbot Laboratories, Chicago, IL), because these triglycerides can be directly absorbed from the small intestine without requiring bile acids. Formulas can be concentrated or have additional carbohydrates or fats added to increase energy density. Oral feeding is the preferred route of formula intake; however, if patients are unable to ingest the needed calories, nasogastric tube drip feedings should be initiated, generally overnight. Fat-soluble vitamins should be supplemented in all cholestatic infants, and blood levels should be routinely monitored to guide dosing. No single multiple vitamin preparation is adequate for all cholestatic infants; most will need additional vitamins K and E, and many will need vitamins D and A beyond a multiple vitamin preparation (Table 3). Vitamin supplementation should be continued for at least 3 months after resolution of jaundice and blood levels checked once an infant has stopped taking the vitamins.

Screening and Prevention


BA is the most common cause of neonatal cholestasis and progresses to end-stage liver disease in up to 80% of

patients within the rst two decades after birth. Early identication and HPE are essential to establish bile ow and avoid liver transplantation within the rst 2 years. (2) A loss of stool pigmentation (acholic stools) may be one of the earliest clinical indicators of BA and is not confounded by breastfeeding, as is relying solely on the presence of jaundice. Lai et al (37) found that 95% of infants who have BA had acholic stool in early infancy. In Taiwan, a national stool color screening system was implemented in 2004 through which an infant stool color card was placed into the child health booklet given to the mother of every newborn. (38) Mothers were to notify a care provider if the infant had an acholic stool before age 1 month and brought the stool color card into the 1-month health supervision visit to show the provider the color of the stools. This program reduced the average age at diagnosis of BA, increased the national rate of the HPE operation performed before age 60 days, increased the 3-month jaundice-free rate after HPE, and increased the 5-year overall survival rate. This program is being initiated in a number of countries; however, its success is not assured in the US health-care system in which there is no standard 1-month infant health provider visit. Pilot testing of a stool color card program would be of great interest and potential public health benet. In a recent study by Harpavat et al, (10) direct bilirubin and conjugated bilirubin levels that were obtained within the rst 72 hours after birth were retrospectively reviewed from 34 infants who had BA and a number of controls. All direct or conjugated bilirubin levels in the BA infants exceeded laboratory norms and were signicantly higher than those of the control subjects (P < .0001). However, total bilirubin remained below the American Academy of Pediatrics recommended

Table 3.

Fat-Soluble Vitamin Supplementation in the Cholestatic Infant


Laboratory Sign of Deciency Retinol: retinol-binding protein <0.8 mol/mol 25-Hydroxyvitamin D <14 ng/mL [ deciency; <30 ng/mL [ insufciency Prolonged prothrombin time, elevated protein in vitamin K absence Vitamin E: total serum lipid ratio <0.6 mg/g Clinical Sign of Deciency Xerophthalmia, keratomalacia Rickets, osteomalacia Coagulopathy Neurologic changes, hemolysis Treatment/Prevention Vitamin A: 3,00010,000 U/d Cholecalciferol: 8005,000 IU/d; 1,25 OH2 cholecalciferol: 0.050.2 mg/kg per d Phytonadione: 2.55 mg twice a week to every day TPGS: 1525 U/kg per d; D-a tocopherol: up to 100 U/kg per d

Vitamin Vitamin A Vitamin D Vitamin K Vitamin E

TPGSD-a-tocopheryl polyethylene glycol 1,000 succinate.

e70 NeoReviews Vol.14 No.2 February 2013

gastrointestinal disorders

cholestasis

phototherapy levels, (39)(40) and the ratio of direct bilirubin:total bilirubin was less than 0.2, the current level at which the North American Society for Pediatric Gastroenterology, Hepatology and Nutrition recommends further evaluation. (12) Additional studies will be needed to conrm these ndings; however, this study suggests that if all newborns were to be screened for elevated direct bilirubin levels in the rst 96 hours after birth regardless of clinical appearance, that it might be possible to identify those who have BA and cholestasis at a young age, potentially improving the outcomes for BA and possibly other conditions. Of course, a cost-effectiveness analysis would need to be conducted to determine the rate of falsepositive ndings and the costs of such a recommendation for essentially universal screening of total and direct or conjugated bilirubin levels before a newborn is discharged from the hospital. Currently, the American Academy of Pediatrics does recommend obtaining a total serum bilirubin or transcutaneous bilirubin level in all newborns before discharge from the hospital.

hepatologist is essential to successful treatment and optimal prognosis. Delayed diagnosis of neonatal cholestasis (and particularly of BA) remains a problem. Further investigation and development of evidence will be necessary to determine if a reliable and cost-effective method of universal screening for neonatal cholestasis should be implemented in the United States. FUNDING: This research was supported in part by National Institutes of Health grants T32 DK067009, U01DK062453, and UL1TR000154.

References
1. Kelly DA, Stanton A. Jaundice in babies: implications for community screening for biliary atresia. BMJ. 1995;310(6988):11721173

2. Serinet MO, Wildhaber BE, Brou P, et al. Impact of age at Kasai


operation on its results in late childhood and adolescence: a rational basis for biliary atresia screening. Pediatrics. 2009;123(5):12801286 3. Hussein M, Howard ER, Mieli-Vergani G, Mowat AP. Jaundice at 14 days of age: exclude biliary atresia. Arch Dis Child. 1991;66 (10):11771179 4. Mieli-Vergani G, Howard ER, Portman B, Mowat AP. Late referral for biliary atresiamissed opportunities for effective surgery. Lancet. 1989;1(8635):421423 5. Lee WS. Pre-admission consultation and late referral in infants with neonatal cholestasis. J Paediatr Child Health. 2008;44(12):5761 6. Balistreri WF. Neonatal cholestasis. J Pediatr. 1985;106(2): 171184 7. Suchy FJ. Neonatal cholestasis. Pediatr Rev. 2004;25(11):388396 8. Balistreri WF, Bezerra JA. Whatever happened to neonatal hepatitis? Clin Liver Dis. 2006;10(1):2753, v 9. Dick MC, Mowat AP. Hepatitis syndrome in infancyan epidemiological survey with 10 year follow up. Arch Dis Child. 1985;60(6):512516 10. Harpavat S, Finegold MJ, Karpen SJ. Patients with biliary atresia have elevated direct/conjugated bilirubin levels shortly after birth. Pediatrics. 2011;128(6):e1428e1433 11. Davis AR, Rosenthal P, Escobar GJ, Newman TB. Interpreting conjugated bilirubin levels in newborns. J Pediatr. 2011;158(4): 562.e1565.e1 12. Bamshad MJ et al. Exome sequencing as a tool for Mendelian disease gene discovery. Genetics. 2011;12(11):745755 13. Moyer V, Freese DK, Whitington PF, et al; North American Society for Pediatric Gastroenterology, Hepatology and Nutrition. Guideline for the evaluation of cholestatic jaundice in infants: recommendations of the North American Society for Pediatric Gastroenterology, Hepatology and Nutrition. J Pediatr Gastroenterol Nutr. 2004;39(2):115128 14. Nievelstein RA, Robben SG, Blickman JG. Hepatobiliary and pancreatic imaging in childrentechniques and an overview of non-neoplastic disease entities. Pediatr Radiol. 2011;41(1):5575 15. Gilmour SM, Hershkop M, Reifen R, Gilday D, Roberts EA. Outcome of hepatobiliary scanning in neonatal hepatitis syndrome. J Nucl Med. 1997;38(8):12791282 16. Esmaili J, Izadyar S, Karegar I, Gholamrezanezhad A. Biliary atresia in infants with prolonged cholestatic jaundice: diagnostic accuracy of hepatobiliary scintigraphy. Abdom Imaging. 2007;32 (2):243247
NeoReviews Vol.14 No.2 February 2013 e71

Conclusions
Cholestatic jaundice, dened as conjugated hyperbilirubinemia, must be considered in any infant presenting with prolonged jaundice longer than 2 weeks (or with hepatomegaly, failure to thrive, acholic stools, or dark urine before or after age 2 weeks) because it can be the rst sign of liver disease. Early detection of cholestasis and subsequent prompt diagnostic evaluation by a pediatric

American Board of Pediatrics NeonatalPerinatal Content Specications


Recognize the association of cholestasis with total parenteral nutrition, know how to manage this, and understand how to diagnose other possible causes. Know the clinical manifestations, diagnostic features, and treatment of infants who have choledochal cysts. Know the pathogenesis and clinical manifestations of extrahepatic biliary atresia. Know the clinical, laboratory, and diagnostic features of extrahepatic biliary atresia that differentiate it from neonatal hepatitis and other causes of cholestasis in the neonate and know the approach to management of extrahepatic biliary atresia. Know the etiology, clinical manifestations, and differential diagnosis of metabolic and familial causes of cholestasis in the neonate. Know the laboratory and imaging features and management of metabolic and familial causes of cholestasis in the neonate.

gastrointestinal disorders

cholestasis

17. Zerbini MC, Gallucci SD, Maezono R, et al. Liver biopsy in neonatal cholestasis: a review on statistical grounds. Mod Pathol. 1997;10(8):793799 18. Russo P, Magee JC, Boitnott J, et al. Design and validation of the biliary atresia research consortium histologic assessment system for cholestasis in infancy. Clin Gastroenterol Hepatol. 2011;9(4): 357.e2362.e2 19. Schreiber RA, Barker CC, Roberts EA, et al. Biliary atresia: the Canadian experience. J Pediatr. 2007;151(6):659665, 665.e1 20. Nio M, Ohi R, Miyano T, Saeki M, Shiraki K, Tanaka K; Japanese Biliary Atresia Registry. Five- and 10-year survival rates after surgery for biliary atresia: a report from the Japanese Biliary Atresia Registry. J Pediatr Surg. 2003;38(7):9971000 21. Lykavieris P, Chardot C, Sokhn M, Gauthier F, Valayer J, Bernard O. Outcome in adulthood of biliary atresia: a study of 63 patients who survived for over 20 years with their native liver. Hepatology. 2005;41(2):366371 22. Sveger T. Liver disease in alpha1-antitrypsin deciency detected by screening of 200,000 infants. N Engl J Med. 1976;294 (24):13161321 23. Emerick KM, Rand EB, Goldmuntz E, Krantz ID, Spinner NB, Piccoli DA. Features of Alagille syndrome in 92 patients: frequency and relation to prognosis. Hepatology. 1999;29(3):822829 24. Hoffenberg EJ, Narkewicz MR, Sondheimer JM, Smith DJ, Silverman A, Sokol RJ. Outcome of syndromic paucity of interlobular bile ducts (Alagille syndrome) with onset of cholestasis in infancy. J Pediatr. 1995;127(2):220224 25. Javid PJ, Malone FR, Dick AA, et al. A contemporary analysis of parenteral nutrition-associated liver disease in surgical infants. J Pediatr Surg. 2011;46(10):19131917 26. Christensen RD, Henry E, Wiedmeier SE, Burnett J, Lambert DK. Identifying patients, on the rst day of life, at high-risk of developing parenteral nutrition-associated liver disease. J Perinatol. 2007;27(5):284290 27. Hoang V, Sills J, Chandler M, Busalani E, Clifton-Koeppel R, Modanlou HD. Percutaneously inserted central catheter for total parenteral nutrition in neonates: complications rates related to upper versus lower extremity insertion. Pediatrics. 2008;121(5):e1152e1159 28. Spencer AU, Neaga A, West B, et al. Pediatric short bowel syndrome: redening predictors of success. Ann Surg. 2005;242 (3):403409, discussion 409412 29. Cober MP, Killu G, Brattain A, Welch KB, Kunisaki SM, Teitelbaum DH. Intravenous fat emulsions reduction for patients

with parenteral nutrition-associated liver disease. J Pediatr. 2012; 160(3):421427 30. Rangel SJ, Calkins CM, Cowles RA, et al; 2011 American Pediatric Surgical Association Outcomes and Clinical Trials Committee. Parenteral nutrition-associated cholestasis: an American Pediatric Surgical Association Outcomes and Clinical Trials Committee systematic review. J Pediatr Surg. 2012;47(1):225240 31. de Meijer VE, Gura KM, Le HD, Meisel JA, Puder M. Fish oilbased lipid emulsions prevent and reverse parenteral nutritionassociated liver disease: the Boston experience. JPEN J Parenter Enteral Nutr. 2009;33(5):541547 32. Soden JS, Lovell MA, Brown K, et al. Failure of resolution of portal brosis during omega-3 fatty acid lipid emulsion therapy in two patients with irreversible intestinal failure. J Pediatr. 2010;156 (2):327331 33. Goulet O, Antbi H, Wolf C, et al. A new intravenous fat emulsion containing soybean oil, medium-chain triglycerides, olive oil, and sh oil: a single-center, double-blind randomized study on efcacy and safety in pediatric patients receiving home parenteral nutrition. JPEN J Parenter Enteral Nutr. 2010;34(5):485495 34. Ng PC, Lee CH, Wong SP, et al. High-dose oral erythromycin decreased the incidence of parenteral nutrition-associated cholestasis in preterm infants. Gastroenterology. 2007;132(5):17261739 35. Kurbegov AC, Setchell KD, Haas JE, et al. Biliary diversion for progressive familial intrahepatic cholestasis: improved liver morphology and bile acid prole. Gastroenterology. 2003;125(4):12271234 36. Whitington PF, Whitington GL. Partial external diversion of bile for the treatment of intractable pruritus associated with intrahepatic cholestasis. Gastroenterology. 1988;95(1):130136 37. Lai MW, Chang MH, Hsu SC, et al. Differential diagnosis of extrahepatic biliary atresia from neonatal hepatitis: a prospective study. J Pediatr Gastroenterol Nutr. 1994;18(2):121127 38. Lien TH, Chang MH, Wu JF, et al; Taiwan Infant Stool Color Card Study Group. Effects of the infant stool color card screening program on 5-year outcome of biliary atresia in Taiwan. Hepatology. 2011;53(1):202208 39. American Academy of Pediatrics Subcommittee on Hyperbilirubinemia. Management of hyperbilirubinemia in the newborn infant 35 or more weeks of gestation. Pediatrics. 2004;114(1):297316 40. US Preventive Services Task Force. Screening of infants for hyperbilirubinemia to prevent chronic bilirubin encephalopathy: US Preventive Services Task Force recommendation statement. Pediatrics. 2009;124(4):11721177

NeoReviews Quiz New minimum performance level requirements


Per the 2010 revision of the American Medical Association (AMA) Physicians Recognition Award (PRA) and credit system, a minimum performance level must be established on enduring material and journal-based CME activities that are certied for AMA PRA Category 1 CreditTM. In order to successfully complete 2013 NeoReviews articles for AMA PRA Category 1 CreditTM, learners must demonstrate a minimum performance level of 60% or higher on this assessment, which measures achievement of the educational purpose and/or objectives of this activity. In NeoReviews, AMA PRA Category 1 CreditTM can be claimed only if 60% or more of the questions are answered correctly. If you score less than 60% on the assessment, you will be given additional opportunities to answer questions until an overall 60% or greater score is achieved.

1. A previously well infant presents with jaundice at age 3 weeks. Results of a blood test reveal a direct bilirubin level of 4 mg/dL and a total bilirubin level of 8 mg/dL. Which of the following is most likely to be true? A. Because the infant is just 3 weeks old, the rst step can be to discontinue human milk and monitor the patient closely for several days to see if the skin color improves.

e72 NeoReviews Vol.14 No.2 February 2013

gastrointestinal disorders

cholestasis

B. Direct hyperbilirubinemia at this age may be due to infectious, metabolic, or anatomic abnormalities, and depending on the clinical context, a comprehensive diagnostic evaluation should be performed expeditiously. C. In virtually all cases, surgery will be necessary. If the infant is growing well and developmentally normal, surgery should be delayed until 6 months to reduce the risks of anesthesia. D. Metabolic conditions should be ruled out before diagnostic tests for anatomic conditions, because a wellappearing patient is unlikely to have biliary atresia. E. The rst step should be intensive phototherapy, after which further diagnostic tests can be performed. 2. A 2-month-old male infant born at term is diagnosed with biliary atresia. You are counseling the parents about the condition. Which of the following is true regarding long-term prognosis for biliary atresia? A. Although liver transplantation may be necessary for w50% of patients, it will most likely be in the third or fourth decade after birth. B. If the child undergoes hepatic portoenterostomy at this age, he is unlikely to have any long-term complications. C. More than 80% of patients who have biliary atresia have other congenital anomalies, which may have more of an impact on survival and disability than liver disease. D. The level of jaundice after hepatic portoenterostomy may help to guide the necessity and timing of future liver transplantation. E. Watchful waiting is an alternative to surgery because some patients may develop a tolerance for conjugated bilirubin. 3. An 8-week-old female presents with jaundice and acholic stools. On further evaluation, the patient is noted to have peripheral pulmonic stenosis on echocardiogram and posterior embryotoxon on eye examination. Which of the following tests is likely to have a positive result in this patient? A. Abnormal sweat chloride test. B. Absence of spleen found on abdominal ultrasound. C. Abnormal protease inhibitor phenotype. D. Mutation in the JAGGED 1 gene. E. Normal g-glutamyl transpeptidase levels. 4. A 28-weeks-gestational-age male is now 3 weeks old. Due to necrotizing enterocolitis, his nutrition has primarily been by parenteral nutrition, which he continues to receive via a peripherally inserted central catheter line. On routine laboratory testing, he is noted to have conjugated hyperbilirubinemia with a direct bilirubin level of 3.4 mg/dL. Which of the following is one of the facets of optimal nutrition therapy for this patient at this time? A. Continue full parenteral nutrition and start phenobarbital intravenously daily, and monitor direct bilirubin level twice weekly. B. Continue parenteral nutrition but do not give intravenous lipids until the direct bilirubin level decreases to less than 2 mg/dL. C. Discontinue parenteral nutrition and give intravenous uids with only dextrose, sodium chloride, and potassium chloride until full enteral feeds are achieved. D. Reduce manganese and copper in the parenteral nutrition solution and monitor levels. E. Switch to a soybean oil lipid emulsion. 5. The 28-weeks-gestational age male who has a history of necrotizing enterocolitis and parenteral nutrition associated cholestasis is now 2 months old and receiving full enteral feedings but continues to have a direct bilirubin level of 3.4 mg/dL. Which of the following is an appropriate aspect of his current nutrition regimen? A. B. C. D. An infant formula with no lipids should be given every other day. Caloric intake should be increased to w110% of the typically recommended allowance. He will require additional vitamins A, D, E, and K to meet nutritional needs. If the infant receives vitamin supplementation, he should stop supplementation w1 week after jaundice is resolved. E. Vitamins should be avoided until the direct bilirubin level is less than 2 mg/dL.
NeoReviews Vol.14 No.2 February 2013 e73

Neonatal Cholestasis Amy G. Feldman and Ronald J. Sokol Neoreviews 2013;14;e63 DOI: 10.1542/neo.14-2-e63

Updated Information & Services References

including high resolution figures, can be found at: http://neoreviews.aappublications.org/content/14/2/e63 This article cites 40 articles, 12 of which you can access for free at: http://neoreviews.aappublications.org/content/14/2/e63#BIBL This article, along with others on similar topics, appears in the following collection(s): Fetus and Newborn Infant http://neoreviews.aappublications.org/cgi/collection/fetus_newb orn_infant Information about reproducing this article in parts (figures, tables) or in its entirety can be found online at: /site/misc/Permissions.xhtml Information about ordering reprints can be found online: /site/misc/reprints.xhtml

Subspecialty Collections

Permissions & Licensing

Reprints

Neonatal Thrombocytopenia Karen S. Fernndez and Pedro de Alarcn Neoreviews 2013;14;e74 DOI: 10.1542/neo.14-2-e74

The online version of this article, along with updated information and services, is located on the World Wide Web at: http://neoreviews.aappublications.org/content/14/2/e74

Neoreviews is the official journal of the American Academy of Pediatrics. A monthly publication, it has been published continuously since . Neoreviews is owned, published, and trademarked by the American Academy of Pediatrics, 141 Northwest Point Boulevard, Elk Grove Village, Illinois, 60007. Copyright 2013 by the American Academy of Pediatrics. All rights reserved. Print ISSN: .

Article

disorders of blood

Neonatal Thrombocytopenia
ndez, MD,* Karen S. Ferna n, MD Pedro de Alarco

Educational Gaps
1. Knowing the differential diagnosis and most likely etiologies of thrombocytopenia in the neonate will lead to more appropriate diagnostic evaluations and treatments. 2. Thrombocytopenia may be a symptom of a variety of congenital or acquired conditions in the neonatal period and prompt further diagnostic evaluations.

Author Disclosure ndez and de Drs Ferna n have disclosed Alarco no nancial relationships relevant to this article. This commentary does contain a discussion of an unapproved/ investigative use of a commercial product/ device.

Abstract
Thrombocytopenia is one of the most common hematologic problems in the neonate. It affects up to 30% of all patients admitted to the neonatal intensive care unit (NICU). The causes of thrombocytopenia in neonates are diverse and include immune, inherited, and acquired disorders. The evaluation of the neonate with thrombocytopenia may be challenging. Developing a diagnostic strategy to evaluate the neonate with thrombocytopenia is key for the practicing clinician. Here, we provide a practical approach to the evaluation of the neonate with thrombocytopenia and an overview of its most common etiologies.

Objectives
1. Describe the differences between neonatal and adult thrombopoiesis. 2. Provide a differential diagnosis of neonatal thrombocytopenia. 3. Describe the clinical presentation and management of the most common forms of thrombocytopenia during the neonatal period. 4. Explain and contrast the pathophysiology of neonatal autoimmune thrombocytopenia and alloimmune thrombocytopenia. 5. Describe the most common inherited causes of thrombocytopenia in the newborn. 6. Discuss the approach to treatment for the neonate with thrombocytopenia according to the etiology.

Abbreviations:
BSS: CAMT: DIC: FA: GP: HPA: ICH: IVIG: NAIT: NEC: NICU: TAR: Tpo: WAS:

Bernard Soulier syndrome congenital amegakaryocytic thrombocytopenia disseminated intravascular coagulation Fanconi anemia glycoprotein human platelet antigen intracranial hemorrhage intravenous immunoglobulin neonatal alloimmune thrombocytopenia necrotizing enterocolitis neonatal intensive care unit thrombocytopenia absent radii thrombopoietin Wiskott-Aldrich syndrome

Denition and Incidence


Thrombocytopenia is dened as a platelet count under 150,000/mL. However, healthy neonates tend to have platelet counts in the range of 100 to 150,000/mL. Thrombocytopenia occurs in less than 1% of all neonates, but is one of the most common hematological problems in neonates, affecting 25% to 30% of all admissions to the neonatal intensive care unit (NICU). The evaluation and management of thrombocytopenia is a challenge for the neonatologist and hematologist, because it can be caused by multiple disease processes. Therefore, a review of the differential diagnosis, the pathophysiology, and the management of the newborn with thrombocytopenia is important.

*Assistant Professor of Pediatrics, University of Illinois College of Medicine at Peoria and Childrens Hospital of Illinois, Peoria, IL. William H. Albers Professor and Chair Department of Pediatrics, University of Illinois College of Medicine at Peoria and Childrens Hospital of Illinois, Peoria, IL.

e74 NeoReviews Vol.14 No.2 February 2013

disorders of blood thrombocytopenia

Platelets Production in the Newborn


Platelets are tiny cellular fragments produced by megakaryocytes. Platelet production, or thrombopoiesis, is a complex process that consists of four main steps: 1. Production of thrombopoietin (Tpo) as the thrombopoietic stimulus. 2. Generation and proliferation of megakaryocyte progenitors. 3. Maturation of megakaryocytes characterized by a progressive increase in nuclear ploidy (the number of sets of chromosomes in a given cell) and cytoplasmic maturity that leads to the generation of large polyploid (8N64N) megakaryocytes. 4. Platelet formation and release of new platelets into the circulation. These mechanisms are signicantly different between neonates and adults with thrombocytopenia. In neonates Tpo levels are not as high in thrombocytopenic neonates, particularly in the small for gestational age infants, as those found in children or adults. The number of megakaryocyte progenitors circulating in the peripheral blood of neonates is higher than in children and adults. They give rise to colonies with a greater number of megakaryocytes, and may be more sensitive to Tpo stimulation in comparison with those of children or adults. Megakaryocytes in the neonate are smaller and have lower ploidy, but their cytoplasm reects that of a mature cell. Last, Tpo effect inhibits megakaryocyte polyploidization in the neonate. Neonates maintain normal platelet counts on the basis of the increased proliferative potential of their megakaryocyte progenitors. Most cases of thrombocytopenia encountered in the NICU are nonimmune and associated with several common neonatal conditions, such as chronic intrauterine hypoxia, sepsis, necrotizing enterocolitis (NEC), and viral infections. Although an in-depth review of the particular mechanism underlying the etiology of these nonimmune causes of thrombocytopenia in the neonate is beyond the scope of this review, understanding the differences between neonatal and adult megakaryopoiesis helps us see what predisposes the neonate to develop thrombocytopenia and the potential utility of thrombopoietic growth factors as potential therapeutic interventions in selected neonates.

maintain the integrity of the vascular endothelium and to control hemorrhage from small-vessel injury through the formation of small aggregates or plugs in the microcirculation. Bleeding tendency results when platelets are decient in number or function. The normal platelet count in newborns and infants is considered to be between 150,000 and 450,000/mL. However, this range of platelet count comes from a limited number of small sample studies of healthy newborns. A study from 18 hospitals in the United States using data from more than 47,000 neonates reported lower limit of platelet ranges from neonates at various gestational ages during their rst 90 days after birth of 123,000/mL in late preterm and term neonates and 104,000/mL in infants of less than 32 weeks gestation. Whether or not a platelet count below 150,000/mL is considered abnormal, it should always be interpreted within the context of the clinical situation. The tendency to bleed is proportional to the number of platelets within the circulation. As such, there is no risk of bleeding with platelet counts greater than 100,000/ mL, minimal or mild risk of bleeding occurs with platelet counts between 20,000 and 100,000/mL, the risk is moderate with platelet counts below 20,000/mL, and the risk is severe and/or there is spontaneous bleeding with platelets below 5,000/mL. In the neonate, the correlation of platelet count with bleeding has not been established. The trauma and the stress of birth can precipitate, although rarely, intracranial or internal bleeding when platelets are below 30,000/mL, and, therefore, clinicians act upon this platelet count to prevent bleeding in the NICU setting. This threshold seems to be higher for preterm infants. Many centers will use a platelet count of 50,000/mL to transfuse preterm infants. However, there is little evidence that this approach will prevent intracranial hemorrhage (ICH). Prospective studies are warranted to establish the most safe and cost-effective threshold at which to transfuse premature infants. The bleeding pattern in the presence of moderate to severe thrombocytopenia is mucocutaneous. The presence of petechiae, bruises, or bleeding from the mucous membranes is characteristic of low platelet counts. In the neonate, intraventricular hemorrhage or intracranial bleed are also possible in the setting of thrombocytopenia.

Platelet Count and Risk of Bleeding


Circulating platelets are about one fth of the diameter of a red blood cell, with a mean volume between 7 and 9 fL. Platelets live for a very short time in the circulation with a half-life of 7 to 10 days. Their primary function is to

Diagnostic Approach to Neonatal Thrombocytopenia


Multiple disease processes can cause thrombocytopenia. A practical approach to the diagnosis and management of thrombocytopenia in the neonate can be based on the time of onset of thrombocytopenia (early 72 hours
NeoReviews Vol.14 No.2 February 2013 e75

disorders of blood thrombocytopenia

after birth or late 72 hours after birth), gestational age of the patient (term versus preterm), on the underlying mechanism (consumption, increased destruction, decreased production), or on whether the thrombocytopenia is due to maternal or infant factors or individualized to the particular infant. Critical parameters that are common to all these approaches include the severity of thrombocytopenia, the maternal history, the health status of the infant, and the presence or absence of congenital malformations. A simplied approach to the diagnosis of thrombocytopenia in the newborn is presented here. It is based on an algorithm (see Figure) and a table (see Table) that takes the above factors into account, especially the severity of thrombocytopenia and the level of illness of the neonate. The sick newborn may become thrombocytopenic from a variety of neonatal complications such as infection, asphyxia, meconium aspiration, respiratory distress syndrome, polycythemia, NEC, and the presence of an indwelling umbilical catheter. In the sick newborn who has severe thrombocytopenia, specic treatment for the underlying condition should be provided and the thrombocytopenia treated symptomatically. In general, mild to moderate thrombocytopenia with a platelet count between 50,000 and 149,000/mL in

a healthy newborn that occurs in the rst 72 hours after birth is associated with a maternal history of placental insufciency. These infants will recover a normal platelet count within 10 days and require only close observation. In sick newborns without evidence of placental insufciency, evaluation for sepsis is warranted in addition to initiation of broad spectrum antibiotics. When the newborns underlying condition improves, the thrombocytopenia should also improve, usually within 5 to 7 days. Persistent thrombocytopenia should alert the physician to look for other causes. Patients with severe thrombocytopenia or a platelet count lower than 50,000/mL should be evaluated for sepsis, disseminated intravascular coagulation (DIC), or neonatal alloimmune thrombocytopenia (NAIT). If any of those are present no additional evaluation is required. In newborns without signs of sepsis, additional evaluation must be pursued and must include: (1) maternal history of thrombocytopenia, (2) detailed familial history of thrombocytopenia, (3) detailed physical examination with special attention to the upper extremities, dysmorphic features suggestive of a congenital anomaly or a particular syndrome, such as thrombocytopenia-absent radii syndrome, Fanconi anemia, trisomy 13, 18, 21, or Turner syndrome.

Figure. Diagnostic approach to an infant with thrombocytopenia. NAIT=Neonatal alloimmune thrombocytopenia.


e76 NeoReviews Vol.14 No.2 February 2013

disorders of blood thrombocytopenia

Table.

Specic Illnesses and Patterns Associated With Neonatal Thrombocytopenia


Categories Immune Infectious Subtypes Alloimmune Autoimmune Bacterial Viral Fungal Parasite Placental insufciency DIC Genetic disorders Chromosomal Familial Differential Diagnoses (Where Applicable) Neonatal alloimmune thrombocytopenia Maternal ITP, lupus, other collagen vascular disorder GBS, Gram-negative rods, Staphylococcus, etc. CMV, HSV, HIV, enteroviruses Candida, other Toxoplasmosis Preeclampsia, eclampsia, chronic hypertension Intrauterine growth restriction due to placental insufciency Asphyxia Sepsis Congenital TTP (rare) Trisomy 13, Trisomy 18, Trisomy 21, Turner syndrome, Jacobsen syndrome Macrothrombocytopenias, Wiskott-Aldrich syndrome, X-linked thrombocytopenias, Amegakaryocytic thrombocytopenia, TAR, Fanconi anemiaa Proprionic acidemia, methylmalonic acidemia, etc. Penicillin and derivatives, vancomycin, metronidazole, etc. Phenytoin, phenobarbital RVT, line-associated thrombosis, sagittal sinus thrombosis Kasabach Kasabach-Merritt, hepatic hemangioendothelioma Severity Severe Severe-moderate Variable Variable Severe Variable Mild-moderate Mild-moderate Severe Severe Severe Variable Variable Onset Early Early Variable Usually early Usually early Early Early Early Early Variable Variable Early Early*

Metabolic Medication induced Antibiotics Heparin Anticonvulsants H-2 receptor antagonists Thrombosis Vascular tumor NEC ECMO

Mild-moderate Variable Variable Variable Variable Moderate Moderate Severe-moderate Variable

Variable Late Late Late Late Variable Variable Usually late Variable

Miscellaneous

Most familial thrombocytopenias are present at birth except for Fanconi anemia, which usually does not appear until childhood. GBSGroup B streptococcus; ITPimmune thrombocytopenic purpura; CMVcytomegalovirus; HSVherpes simplex virus; HIVhuman immunodeciency virus; TTPthrombotic thrombocytopenic purpura; TARthrombocytopenia-absent radii syndrome; RVTrenal vein thrombosis; NECnecrotizing enterocolitis; ECMOextra corporeal membrane oxygenation. a Reproduced with permission from Fernndez KS, de Alarcn PA. Congenital thrombocytopenias and thrombocytopathies. In: de Alarcn PA, Christensen RD, Werner EJ, eds. Neonatal Hematology: Pathogenesis, Diagnosis, and Management of Hematologic Problems. 2nd ed. Cambridge, United Kingdom: Cambridge University Press; 2013, p. 174.

In newborns with a negative maternal or familial history and an unrevealing physical examination, other infections, such as toxoplasmosis, other viruses and syphilis, rubella, cytomegalovirus, and human immunodeciency virus complex; human immunodeciency virus infection; or enteroviruses, should be considered. In addition, catheter-related thrombosis, chromosomal anomalies, and inborn errors of metabolism, especially propionic acidemia and methylmalonic acidemia, should be considered.

The most common etiology of severe thrombocytopenia in an otherwise healthy-looking newborn is immunemediated thrombocytopenia in which there is passage of maternal antibodies from the mother to the fetus. Other rarer disorders, such as vascular tumors or hemangiomas with Kasabach-Merritt phenomenon and renal vein thrombosis, should be investigated. When thrombocytopenia occurs more than 72 hours after birth, it is more likely to be due to bacterial or fungal
NeoReviews Vol.14 No.2 February 2013 e77

disorders of blood thrombocytopenia

sepsis and/or NEC. For patients in whom a bacterial or fungal process is excluded as etiology of thrombocytopenia, viral infections such as herpes simplex virus and cytomegalovirus, DIC, catheter-related thrombosis, drug-induced thrombocytopenia, heparin-induced thrombocytopenia, or other inherited disorders should be considered. This group of patients constitutes the most common reason for consultation to the hematology team. Immunemediated thrombocytopenias should also be considered in this group, in particular, because they tend to worsen in the rst few days after birth. The differential diagnosis of immune-mediated thrombocytopenias is between NAIT, maternal autoimmune disorders, or, rarely, maternal drug-induced immune thrombocytopenia. In the following sections we will highlight some of the features of the most common causes of thrombocytopenia in newborns in the three major categories of destruction of platelets by immune mediated process, decreased or abnormal production of platelets owing to inherited disorders, and consumption of platelets related to some acquired disorders.

Immune-Mediated Causes of Neonatal Thrombocytopenia


Neonatal Alloimmune Thrombocytopenia
NAIT is a rare disorder that presents in an otherwise wellappearing newborn with moderate to severe thrombocytopenia. NAIT occurs when fetal platelets contain an antigen inherited from the father, a human platelet antigen (HPA) that the mother lacks. Fetal platelets that cross the placenta into the maternal circulation trigger the production of maternal antiplatelet antibodies against the foreign antigen. These antibodies cross the placenta into the fetal circulation and destroy platelets resulting in fetal and neonatal thrombocytopenia. There are sixteen HPAs identied but only three cause 95% of the NAIT cases (HPA-1a, HPA-5b, and HPA-15b). Feto-maternal incompatibility for HPA-1a is responsible for 75% of cases in whites. HPA-1a incompatibility occurs in 1:350 pregnancies, although thrombocytopenia develops in only 1:1,000 to 1,500 pregnancies. Infants with NAIT will have symptoms of mucocutaneous bleeding but will look otherwise healthy. Typically, platelet count falls below 50,000 in the rst few days after birth, but then rises as the alloantibody concentration declines, which usually happens in 1 to 4 weeks, the expected half-life of immunoglobulin G. This immunemediated platelet disorder is the equivalent to Rh sensitization of red blood cells with the only difference that NAIT often develops in the rst pregnancy of an at-risk
e78 NeoReviews Vol.14 No.2 February 2013

couple. The most serious complication of NAIT is ICH. This may occur in as many as 10% of the cases, and up to 50% of the time it happens before birth. All infants with severe thrombocytopenia due to NAIT should have a cranial ultrasound to look for evidence of ICH. If available, the best alternative to treat these patients is the use of HPA-1anegative platelets from the blood bank. Random donor platelets, even though they are likely to have the target antigen, are effective in treating the infant with severe thrombocytopenia. Platelets obtained from the mother are also effective, but these platelets need to be washed to minimize the presence of the circulating anti-HPA-1a antibodies. Specic platelet antigen and antibody testing are not readily available at all centers but can be requested at major referral laboratories. The administration of intravenous immunoglobulin (IVIG) may be helpful and represents another alternative treatment but less effective than platelet transfusions. Once an infant is diagnosed with NAIT, it is important to carry out a complete diagnostic plan with genotyping of mother and father to be able to provide genetic counseling. In subsequent pregnancies, if the father is a homozygote for the affected antigen, therapy should be started as early as the 13th week of pregnancy with weekly IVIG with or without prednisone, depending on the severity of the previously affected infant or if there was a history of ICH. If the father is a heterozygote for the antigen, the risk of the infant must be determined by molecular analysis of fetal cells circulating in the maternal blood, if available, or by invasive procedures: chorionic villi biopsy or amniocentesis, which both carry signicant risks to the pregnancies, to establish if the fetus is affected.

Neonatal Autoimmune Thrombocytopenia


Neonatal autoimmune thrombocytopenia, in contrast to NAIT, is caused by the passage of maternal antibodies that react with both maternal and infant platelets, and therefore both mother and infant are affected. This disorder occurs in maternal autoimmune disorders such as immune thrombocytopenic purpura or systemic lupus erythematosus. It occurs in 1 to 2:1,000 pregnancies. The diagnosis becomes obvious from a maternal history of thrombocytopenia. Transplacental passage of maternal autoantibodies in this setting is much less of a clinical problem than NAIT. It is important to note that the maternal history is not always positive, because there are many thrombocytopenic mothers who would be asymptomatic and therefore unaware of their own disorder. All neonates of mothers with autoimmune diseases should have their platelet count determined at birth. In most

disorders of blood thrombocytopenia

cases, the platelet count rises spontaneously by day 7. In cases of severe thrombocytopenia, treatment with IVIG is recommended. The presence of unexplained thrombocytopenia in a newborn that is suggestive of autoimmune destruction and in whom NAIT has been excluded should trigger evaluation for the presence of an autoimmune disorder in the mother, because neonatal thrombocytopenia can sometimes be the presenting sign of maternal disease.

Inherited Thrombocytopenias
Aneuploidies
Thrombocytopenia is seen in neonates with trisomy 13, 18, and 21, and also with Turner syndrome. The exact mechanism of thrombocytopenia is unknown, but may be due to reduced platelet production and the pathogenesis may be similar to that seen in chronic fetal hypoxia.

eczema, and recurrent bacterial and viral infections. Most cases will not present during the neonatal period unless there is a known family history. The disorder usually presents in the rst year after birth with bleeding symptoms. Bleeding is due to abnormal platelet function, reduced platelet survival, and thrombocytopenia. X-linked thrombocytopenia is a part of the spectrum of WAS. These patients have thrombocytopenia and small platelets as the major manifestations of their disease. However, the difference is in the variable and less severe degrees of eczema and immunodeciency.

Fanconi Anemia
Fanconi anemia (FA) can present in the newborn with persistent thrombocytopenia. It is, in most cases, an autosomal recessive disorder. The infant may present with thrombocytopenia alone, pancytopenia, or with dysmorphic features only. The associated congenital abnormalities of hypopigmented and hyperpigmented skin lesions, microcephaly, small size, urinary-tract abnormalities, and upper-extremity radial-side abnormalities involving the thumb, should alert the physician to the possibility of FA. The cross-linking agent diepoxybutane has been used effectively to diagnose FA and is the standard diagnostic test. Treatment is rarely necessary in the neonatal period.

Bernard-Soulier Syndrome
Bernard-Soulier syndrome (BSS) is a moderate to severe platelet function defect characterized by mild thrombocytopenia, giant platelets, and mucosal type bleeding. It is a very rare disorder with an incidence of 1:1,000,000. It may present in the neonatal period, although bleeding is not usually severe. It is a qualitative platelet disorder with a defect in the von Willebrand factor receptor, the glycoprotein (GP) complex GP IbIXV. The defect in BSS is in the GPIba gene, and also in the GPIbb and GPIX genes, mapped to chromosome 17, chromosome 22, and chromosome 3, respectively. Defect in chromosome 22 and abnormality in GPIbb explains why infants with DiGeorge syndrome and cardiac disease may develop severe bleeding due to a coexisting BSS. The diagnosis of BSS is conrmed by the absence of CD41a or PGPIbIXV by ow cytometry. Treatment for BSS is mostly supportive and with platelet transfusion for life-threatening bleeding. Mothers with BSS may develop alloantibodies against GPIbIXV that can cross the placenta, and, therefore, their offspring can develop NAIT due to the passage of alloantibodies against GPIbIXV antigens.

Thrombocytopenia Absent Radii Syndrome


This syndrome is characterized by the bilateral absence of the radii and thrombocytopenia. The inheritance pattern of the thrombocytopenia absent radii (TAR) syndrome is autosomal recessive. Both boys and girls are affected, but there is a predominance of girls. The onset of symptoms usually occurs very early in life. Half of the patients have onset of hemorrhagic manifestations in the rst week after birth, and most develop thrombocytopenia by 4 months of age. In contrast to infants with FA, in patients with TAR syndrome both thumbs are present. The prognosis in the TAR syndrome is dependent on the severity of the hemorrhagic manifestations. If the patient survives the rst year after birth, the hemorrhagic manifestations resolve, because the platelet count spontaneously improves to low normal levels which are then maintained. Treatment is supportive with platelet transfusions indicated only in the event of active bleeding.

Wiskott-Aldrich Syndrome
Small platelets on peripheral blood lm and/or a low mean platelet volume may indicate WiskottAldrich syndrome (WAS). WAS is due to mutations in the WAS protein gene on the short arm of the X chromosome. Mutations in this gene have been isolated to Xp11.23. WAS is characterized by microthrombocytopenia,

Congenital Amegakaryocytic Thrombocytopenia


Congenital amegakaryocytic thrombocytopenia (CAMT) is a rare recessive autosomal disorder presenting during the neonatal period with thrombocytopenia. Most affected infants have petechiae or other evidence of
NeoReviews Vol.14 No.2 February 2013 e79

disorders of blood thrombocytopenia

bleeding. Physical anomalies are present in approximately 50% of the patients. CAMT typically presents with isolated thrombocytopenia; however, 50% of the patients will later progress to aplastic anemia during infancy or early childhood. The bone marrow shows decreased to absent megakaryocytes with normal erythroid and myeloid precursors. Bleeding episodes in neonates with CAMT are treated by platelets transfusions, but stem cell transplantation is the denitive form of therapy for this disorder. There is another variant of amegakaryocytic thrombocytopenia with radioulnar synostosis. Patients with amegakaryocytic thrombocytopenia with radioulnar synostosis have a mutation in the HOXA11 gene that distinguishes them from TAR syndrome, and from CAMT in which the mutation is located in the cMPL gene.

anomalies with visceral involvement. The thrombocytopenia is due to the trapping and consumption of platelets in the endothelium of the abnormal blood vessels. The treatment of these lesions requires supportive treatment with plasma and platelet transfusion if DIC is present. Some of these vascular malformations with aggressive behavior may need treatment with steroids, interferon, vincristine, and other chemotherapy agents. More recently, the use of the angiogenesis inhibitor bevacizumab and the use of the mTOR inhibitor, rapamycin, have shown some activity; however, further studies are necessary before recommending therapy with these agents.

Thrombotic Disorders
Acquired thrombotic events in the NICU have increased over the past several years, mainly because of the high complexity of the patients cared for in the NICU requiring indwelling catheters and being at risk for several factors that predispose them to secondary thrombotic events. The use of heparin ushes to maintain the patency of indwelling catheters is also a risk for the development of heparin-induced thrombocytopenia that is associated with arterial thrombosis. Inherited deciency of ADAMTS13, the cleaving protease of von Willebrand factor, that causes thrombotic thrombocytopenic purpura may present in the newborn period. Renal vein thrombosis should also be considered in the differential diagnosis of patients with thrombocytopenia and renal failure. Thrombocytopenia is part of the clinical presentation of anticoagulant factor deciencies, and these deciencies should be considered in the differential diagnosis of thrombocytopenia. However, the severe form of these deciencies presents with purpura fulminans and diffuse thromboses and not just isolated thrombocytopenia.

Giant Platelet Syndromes


Giant platelet syndromes may present in the neonatal period. The characteristic of these disorders is not only a reduced platelet count, but the appearance of large platelets on the peripheral smear. Several of the rare giant platelet syndromes present in the fetus, including the May-Hegglin anomaly, which is characterized by the presence of leukocyte Dohle-like inclusion bodies. This could be a rare cause of fetal or neonatal ICH. The defect in May-Hegglin anomaly is in the MY-H9 gene on chromosome 22q. This mutation is also found in other giant platelet syndromes such as Fletcher syndrome, where leukocyte inclusions are absent and there is association of sensorial hearing loss, nephritis and cataracts, and Epstein syndrome, in which there are no leukocyte inclusions but there is association to hearing loss and nephritis without cataracts. Other macrothrombocytopenias are a group of heterogeneous disorders that include functional platelet disorders like Bernard-Soulier syndrome that was previously described, gray platelet syndrome in which there is lack of platelet granules, and Jacobsen-Paris-Trousseau syndrome that is associated with psychiatric problems or mental retardation.

Conclusions
Thrombocytopenia is a common problem in the newborn. The differential diagnosis of thrombocytopenia in the neonate can be simplied when taking into account the severity of the thrombocytopenia and the clinical appearance of the neonate. Most episodes of thrombocytopenia in the newborn occur after the rst 72 hours after birth and are most commonly caused by infectious process. Persistent thrombocytopenia, or thrombocytopenia that does not respond to adequate treatment of the presumed etiology of the low platelet count, deserves further investigation to look for some of the rare causes of thrombocytopenia in neonates

Consumptive Causes of Thrombocytopenia in the Neonatal Period


Kasabach-Merritt Phenomenon
This is an important cause of thrombocytopenia in the newborn. It typically presents with profound thrombocytopenia, microangiopathic anemia, and DIC in association with a vascular malformation. The diagnosis is obvious when the vascular anomaly is cutaneous, but it may be more challenging with the presence of vascular
e80 NeoReviews Vol.14 No.2 February 2013

disorders of blood thrombocytopenia

including immune-related disorders, inherited thrombocytopathies and other acquired causes of platelet consumption.

Suggested Reading
Chakravorty S, Roberts I. How I manage neonatal thrombocytopenia. Br J Haematol. 2012;156(2):155162 Drachman JG. Inherited thrombocytopenia: when a low platelet count does not mean ITP. Blood. 2004;103(2):390398 Ferrer-Marin F, Liu ZJ, Gutti R, Sola-Visner M. Neonatal thrombocytopenia and megakaryocytopoiesis. Semin Hematol. 2010;47(3):281288 Hammill AM, Wentzel M, Gupta A, et al. Sirolimus for the treatment of complicated vascular anomalies in children. Pediatr Blood Cancer. 2011;57(6):10181024 Nurden P, Nurden AT. Congenital disorders associated with platelet dysfunctions. Thromb Haemost. 2008;99(2):253263 Risson DC, Davies MW, Williams BA. Review of neonatal alloimmune thrombocytopenia. J Paediatr Child Health. 2012;48(9):816822 Sola-Visner M, Sallmon H, Brown R. New insights into the mechanisms of nonimmune thrombocytopenia in neonates. Semin Perinatol. 2009;33(1):4351

American Board of Pediatrics Neonatal-Perinatal Content Specications


Know the normal pattern of platelet production and maturation. Know the causes and pathophysiology of neonatal thrombocytopenia and thrombocytosis. Know the clinical and laboratory manifestations and management of neonatal thrombocytopenia and thrombocytosis.

NeoReviews Quiz New Minimum Performance Level Requirements


Per the 2010 revision of the American Medical Association (AMA) Physicians Recognition Award (PRA) and credit system, a minimum performance level must be established on enduring material and journal-based CME activities that are certied for AMA PRA Category 1 CreditTM. In order to successfully complete 2013 NeoReviews articles for AMA PRA Category 1 CreditTM, learners must demonstrate a minimum performance level of 60% or higher on this assessment, which measures achievement of the educational purpose and/or objectives of this activity. In NeoReviews, AMA PRA Category 1 CreditTM can be claimed only if 60% or more of the questions are answered correctly. If you score less than 60% on the assessment, you will be given additional opportunities to answer questions until an overall 60% or greater score is achieved.

1. Which of the following distinguishes thrombopoiesis in neonates from adults? a. Megakaryocytes in neonates are larger than those found in adults. b. The mechanisms of neonatal response and physiology of thrombocytopenia are essentially identical to adults. c. The number of megakaryocyte progenitors circulating in peripheral blood of neonates is lower than in adults. d. Thrombopoietin does not stimulate thrombopoiesis in neonates. e. Thrombopoietin levels are not as high in thrombocytopenic neonates, particularly small for gestational age infants, compared with adults. 2. The laboratory calls you regarding a patient in the NICU to note an abnormal platelet count of 105,000/mL. This patient is a 2-day-old 33-weeks-gestational-age female. Which of the following is the most appropriate management decision? a. b. c. d. e. A platelet level and antibody tests should be ordered for the mother. A platelet transfusion of 10 mL/kg should be given. The patient can be monitored clinically, and a platelet count can be repeated the next morning. This patient is at high risk of bleeding, particularly intracranial hemorrhage. The patient should undergo metabolic and genetic screening, including chromosomal disorders.

NeoReviews Vol.14 No.2 February 2013 e81

disorders of blood thrombocytopenia

3. A 38-weeks-gestational-age newborn male has a complete blood count drawn several hours after birth because of being small for gestational age, and thrombocytopenia is noted, with other parameters of the blood count being in normal ranges. In regard to the possibility of sepsis, which of the following factors would lead you to consider close observation and not giving antibiotics at this time? a. The platelet count is 25,000/mL, and the patient has no evidence of bleeding. b. The platelet count is 95,000/mL, and the patient has hypotension and capillary rell greater than 3 seconds. c. The platelet count is 85,000/mL, and the patients mother has a known history of placental insufciency. d. The platelet count is 50,000/mL, the patient has no bleeding symptoms but has respiratory distress requiring oxygen. e. Platelet count should not be considered in the evaluation for sepsis. 4. A 38-weeks-gestational-age male presents in the well infant nursery at 2 days after delivery with mucocutaneous bleeding, and a complete blood count reveals platelet count of 25,000/mL. He appears well otherwise. He is admitted to the NICU and a cranial ultrasound reveals bilateral grade I intraventricular hemorrhage. His mothers platelet count is 175,000/mL. What is the most likely diagnosis? a. b. c. d. e. Bernard-Soulier syndrome. Laboratory error. Neonatal alloimmune thrombocytopenia. Neonatal autoimmune thrombocytopenia. Wiskott-Aldrich syndrome.

5. A 1-kg, 29-weeks-gestational-age female is now 2 weeks old. She is on nasal cannula oxygen and parental nutrition via a peripherally inserted central line, and she is receiving gavage feedings of both maternal breast milk and premature infant formula. There have been increased apnea and bradycardia events over the past 24 hours with an increased need for oxygen concentration. A complete blood count reveals hematocrit of 26%, white blood cell count of 6,000/mL, and platelet count of 65,000/mL. The platelet count 1 day after delivery was noted to be 125,000/mL. What is the most appropriate next step in management of this patient? a. b. c. d. e. Avoid giving any medications for the next 48 hours in case thrombocytopenia may be drug-related. Give 10 mL of packed platelet transfusion over the next hour. Obtain platelet counts from the mother and father. Perform sepsis evaluation and start antibiotics while awaiting cultures. Stop giving maternal breast milk for 48 hours in case there may be passage of maternal drugs via breast milk, and instead use formula only.

e82 NeoReviews Vol.14 No.2 February 2013

Neonatal Thrombocytopenia Karen S. Fernndez and Pedro de Alarcn Neoreviews 2013;14;e74 DOI: 10.1542/neo.14-2-e74

Updated Information & Services References

including high resolution figures, can be found at: http://neoreviews.aappublications.org/content/14/2/e74 This article cites 7 articles, 1 of which you can access for free at: http://neoreviews.aappublications.org/content/14/2/e74#BIBL

Subspecialty Collections

This article, along with others on similar topics, appears in the following collection(s): Fetus and Newborn Infant http://neoreviews.aappublications.org/cgi/collection/fetus_newb orn_infant Information about reproducing this article in parts (figures, tables) or in its entirety can be found online at: /site/misc/Permissions.xhtml Information about ordering reprints can be found online: /site/misc/reprints.xhtml

Permissions & Licensing

Reprints

Hemolytic Disease of the Fetus and Newborn Mary Beth Ross and Pedro de Alarcn Neoreviews 2013;14;e83 DOI: 10.1542/neo.14-2-e83

The online version of this article, along with updated information and services, is located on the World Wide Web at: http://neoreviews.aappublications.org/content/14/2/e83

Neoreviews is the official journal of the American Academy of Pediatrics. A monthly publication, it has been published continuously since . Neoreviews is owned, published, and trademarked by the American Academy of Pediatrics, 141 Northwest Point Boulevard, Elk Grove Village, Illinois, 60007. Copyright 2013 by the American Academy of Pediatrics. All rights reserved. Print ISSN: .

Article

disorders of blood

Hemolytic Disease of the Fetus and Newborn


Mary Beth Ross, MD,* n, MD Pedro de Alarco

Abstract
Hemolytic disease of the fetus and newborn (HDFN) is the result of immune-mediated destruction of fetal or newborn red blood cells when such cells contain antigens that are not present in the maternal blood. HDFN is now the preferred term that replaces the historic term erythroblastosis fetalis. Sensitization of the mother to fetal-newborn red blood cells requires fetomaternal hemorrhage in most cases except in ABO incompatibility where naturally occurring antibodies against A and B antigens are present in mothers with O blood type. The most common antigen involved in HDFN is Rhesus D. Kell 1 HDFN is rare but commonly associated with severe anemia and lower titers of anti-Kell antibodies in maternal serum in severely affected infants. Prevention of Rhesus D HDFN with anti-D immunoglobulin during pregnancy, delivery, and fetal-maternal events that predispose to fetomaternal hemorrhage, have markedly decreased the incidence of the disorder but may not be available in low-income countries. An algorithm is available to manage affected pregnancies by using antibody titers, fetal middle cerebral artery velocities, intrauterine transfusions, and timed delivery. Infants who have mild to moderate anemia may tolerate normal labor, but severely affected infants may require transfusion or exchange transfusions at birth, and the delivery team needs to be prepared. Delayed anemia in the transfused infants is still a concern, and the infants need to be closely followed after delivery. Phototherapy has largely replaced exchange transfusion in the management of hyperbilirubinemia. With appropriate early detection and multidisciplinary planning, infants who have HDFN can be delivered in a timely manner with appropriate planning for postnatal resuscitation and postnatal therapy resulting in good neonatal outcomes.

Author Disclosure Drs Ross and de n have disclosed Alarco no nancial relationships relevant to this article. This commentary does not contain a discussion of an unapproved/ investigative use of a commercial product/ device.

Objectives

After completing this article, readers should be able to:

1. Recognize the symptoms and signs associated with hemolytic disease of the fetus and newborn (HDFN). 2. Understand the pathophysiology of HDFN. 3. List the red blood cell antigens most commonly associated with HDFN. 4. Identify the fetus at risk for HDFN. 5. Discuss the clinical management of fetus/newborn affected by HDFN.

Introduction Abbreviations
FMH: HDFN: IAT: MCA: pRBC: RBC: RhD: fetomaternal hemorrhage hemolytic disease of the fetus and newborn indirect antiglobulin testing middle cerebral artery packed red blood cell red blood cell Rhesus D

Advances in prevention and detection have markedly decreased the incidence of hemolytic disease of the fetus and newborn (HDFN). The disorder, caused by red blood cell (RBC) incompatibility between infant and mother, and its multiple clinical manifestations were rst brought together in 1932 in the landmark article by Dr Louis K. Diamond, where he coined the term erythroblastosis fetalis, based on the morphology of the peripheral blood smear seen in infants who have severe diseases. HDFN is now the preferred term encompassing all infants who have alloimmune

*Assistant Professor of Pediatrics, University of Illinois College of Medicine at Peoria and Childrens Hospital of Illinois, Peoria, IL. William H. Albers Professor and Chair, Department of Pediatrics, University of Illinois College of Medicine at Peoria and Childrens Hospital of Illinois, Peoria, IL.

NeoReviews Vol.14 No.2 February 2013 e83

disorders of blood

hemolytic disease

hemolysis, whether or not erythroblasts are present. The work of Landsteiner and other investigators dened RBC antigens and their role in transfusion reaction by 1940, and Levine et al in 1941 gathered enough cases to document that HDFN was indeed caused by blood group incompatibility between a mother and her infant. HDFN is the result of immune destruction of fetal or newborn RBCs. Maternal antibodies develop when fetal RBCs express cell surface antigens that are not present on the maternal RBCs. For this process to occur, fetal RBCs must enter the maternal circulation secondary to fetomaternal hemorrhage (FMH). The exception to this rule is ABO incompatibility because type O mothers have naturally occurring anti-A and anti-B antibodies. HDFN should be considered in the differential diagnosis of postnatal early, severe, or prolonged jaundice. It also should be considered in the differential diagnosis of neonatal anemia, in particular, if it is severe or associated with hydrops fetalis. The presence of maternal RBC antibodies and/or a positive direct antibody testing in the infant are diagnostic for HDFN. A comprehensive maternal history is essential for the proper diagnosis of HDFN. Particular emphasis should be placed on uncovering maternal events that predispose to FMH. A maternal history of a previous pregnancy, particularly a complicated pregnancy, history of hydrops fetalis, a miscarriage, early termination of pregnancy, blood transfusion, or clinical documentation of FMH, should alert the clinician to the possibility of HDFN in an infant with anemia and/or jaundice. The greatest advances in this disorder have been the introduction of very effective prevention strategies, the aggressive use of phototherapy for jaundice, and the introduction of noninvasive techniques to monitor the affected fetus.

association with acute illness. Family members who have a history of splenectomy unrelated to trauma or early gallbladder disease may suggest an undiagnosed inherited RBC membrane defect. Hemoglobinopathies also may contribute to neonatal hemolysis. Hemoglobinopathy screening in the United States has been available in most states for over a generation, and today all 50 states screen for these disorders and they are diagnosed at birth. Nonetheless, recent immigrants to the United States may have previously undiagnosed hemoglobinopathies. Review of the blood smear, newborn screening, conrmatory hemoglobin electrophoresis, and potentially maternal and paternal hemoglobin electrophoresis should clarify this diagnosis. RBC enzyme defects such as glucose-6-phophate dehydrogenase can lead to hemolysis and anemia. Signicant and prolonged hyperbilirubinemia and even kernicterus have been described in infants who have glucose-6-phophate dehydrogenase deciencies, particularly in the Philippines, Africa, and Greece. Hemolysis can occur from disorders of glycolysis such as pyruvate kinase deciency. Here too, family history and ethnic background can be key factors in identifying the risk for RBC enzyme defects.

Mechanisms of Maternal Exposure


All individuals with blood type O naturally express antibodies to A and B RBC surface antigens. No previous exposure to blood group antigens is necessary for antibody development. These antibodies are immunoglobulin G type and cross the placenta. Individuals who have blood type A have antibodies against type B and vice versa. However, these antibodies are predominantly immunoglobulin M class and do not cross the placental barrier. For all other blood group antigens, maternal exposure to blood group antigens is necessary for the development of antibodies. The most frequent cause of maternal sensitization is FMH. FMH can occur in the rst trimester, but is most common in the third trimester. Numerous fetal events or procedures may be associated with previously underappreciated risk for FMH. See the Table for a listing of events that may be associated with FMH. Women who have a known history of transfusion represent only a small proportion of all pregnant women. However, women who have a previous transfusion history represent half of the pregnancies affected by non-Rhesus D (RhD) HDFN. Routine blood typing and cross-match screen for ABO and RhD blood types, but none of the other blood types involve HDFN. As more children survive previously serious childhood illnesses such as cancer and congenital heart disease, we are likely to see an increase

Differential Diagnosis
The two main signs of HDFN are anemia and hyperbilirubinemia. The anemia is hemolytic and the bone marrow is reactive with reticulocytosis and often presents immature RBCs, erythroblasts, in the peripheral blood; hence, the original term erythroblastosis fetalis was given to the disorder. Other types of nonimmune-mediated hemolysis can result in anemia and hyperbilirubinemia. These include RBC membrane defects such as hereditary spherocytosis and hereditary elliptocytosis. Although these disorders are hereditary, their phenotype is variable, and the RBC defects may go undiagnosed well into adulthood. Clues that can be suggestive of undiagnosed RBC membrane defects include family members who are intermittently jaundiced, intermittently have scleral icterus, or intermittently have dark urine. These episodes generally occur in
e84 NeoReviews Vol.14 No.2 February 2013

disorders of blood

hemolytic disease

Table.

Mechanisms of Maternal Exposure to Red Blood Cell Antigens

1. ABO group Innate antibody production, no previous exposure needed 2. Known fetal-maternal hemorrhage a. Placental abruption b. Other placental bleeding or injury c. Fetal surgery d. In utero transfusion e. Delivery of previous infant f. Delivery of previous infant affected by HDFN 3. Risk for unappreciated fetal-maternal hemorrhage a. Ectopic pregnancy b. Abnormal placental insertion c. Spontaneous abortion d. Induced abortion e. Fetal demise f. Amniocentesis g. Cordocentesis h. Chorionic villus sampling i. Maternal abdominal trauma j. Fetal version maneuvers k. Delivery of previous infant requiring exchange transfusion or phototherapy 4. Known maternal transfusion 5. Maternal history with potential for unappreciated maternal transfusion a. Prolonged hospital stay as an infant b. Survivor of childhood cancer c. Repair of craniosynostosis in childhood d. Correction of congenital heart defect e. Major surgical procedure in childhood or adulthood f. Abdominal surgery in childhood or adulthood g. Splenectomy for unclear indication (may suggest maternal red cell defect) h. Return to operating room within 7 days of delivery of an infant

a family is willing to disclose use of egg donor, sperm donor, or embryo adoption, ABO blood type of the donors may or may not be known to the pregnant woman. This circumstance can set up otherwise uninheritable combinations of RBC antigens. For example, an O (negative) mother with an O (negative) husband could deliver an AB(positive) infant who is at risk for HDFN from ABO and Rh mismatch.

Red Blood Cell Antigens Most Frequently Involved in Hemolytic Disease of the Fetus and Newborn
Rhesus (RhD) incompatibility is the best described cause of HDFN. Despite international efforts aimed at prevention of HDFN in infants of Rh-negative mothers, Rh remains the most commonly identied RBC antigen causing HDFN. RhD-negative denotes the lack of D antigen on the RBC surface. Eleven percent to 35% of white populations are RhD-negative because of gene deletion. In contrast, most East Asian and African populations that lack RBC surface expression of RhD have a grossly intact gene. In Africans, a 37-bp insertion results in the insertion of a stop codon leading to a prematurely shortened protein product. A special circumstance exists with an RhD variant called weak D. There is both an altered protein sequence and decreased cell surface protein expression. Serologic testing will identify this person as RhD-negative. Although more sensitive testing can demonstrate the presence of the protein on the cell surface, importantly, these apparently RhD-negative mothers will not form anti-D antibodies even if they are transfused with RhD-positive blood. The second most common RBC antigen associated with HDFN is the ABO blood group. HDFN due to an ABO blood group mismatch occurs almost exclusively in infants with mothers of blood type O. Hemolysis is more common with anti-A than with anti-B. The clinical presentation for HDFN due to ABO blood group is predominantly hyperbilirubinemia without severe anemia. Phototherapy is generally sufcient for most of these infants. Another major cause of HDFN is Kel1 antigen of the Kell antigen system. Kell is a glycoprotein containing 15 antigens and their antithetical variants. It is the rst erythroid-specic antigen known to be expressed during erythroid development. Clinically, anti-Kel1 HDFN is manifest by more severe anemia and reticulocytopenia. Hyperbilirubinemia is less severe than with other RBC antigens. Anti-Kel1 antibody is rare and is only found in 0.1% of pregnant women. However, most of these women have developed antibody because of previous transfusion exposure. Fortunately, only 9% of people of European
NeoReviews Vol.14 No.2 February 2013 e85

in HDFN from maternal transfusion. In fact, the mother may be unaware of her own blood product exposure, because treatment may have occurred during her early childhood. See the Table for a listing of maternal history elements that raise concern for unappreciated exposure to RBC antigens. The ever expanding utilization of reproductive technologies leads to unexpected risks for HDFN. The battle with infertility is highly personal in nature. That, combined with our fragmented health-care system and highly mobile society, leads to signicant opportunity for utilization of egg donor, sperm donor, or embryo adoption to go undisclosed to the health-care team at the time of delivery. Even when

disorders of blood

hemolytic disease

descent and 2% of people of African descent express Kel1. Because of the early erythroid lineage expression of Kell when Kel1 is present, anti-Kel1 is associated with a lower critical antibody titer (1:8) than anti-RhD or antibodies to other RBC antigens. A variety of other RBC antigens have been described as causing HDFN. For a detailed listing of RBC antigens, the severity of HDFN associated with each antigen, and reference citation, see Table 6.1 in de Alarcon, Werner, and Christensens Neonatal Hematology.

Clinical Management: Prevention of Hemolytic Disease of the Fetus and Newborn Due to Rhesus D
The rst approach to prevention of HDFN due to an RhD-positive fetus born to an RhD-negative mother is administration of one postnatal dose of anti-RhD. Between 1968 and 1983, both maternal immunization and perinatal infant deaths were reduced by 90%. Subsequently, clinical trials were performed demonstrating that the administration anti-RhD at 28 weeks gestation in addition to the postnatal dose could further decrease the frequency of immunization from the remaining 2% down to 0.2%. Similarly, an appreciation rose for other events that led to FMH effectively immunizing the mother further against RhD. We now know that fetal and pregnancy events such as ectopic pregnancy, spontaneous or induced abortion, fetal demise, and maternal abdominal trauma can result in maternal immunization from FMH. Invasive medical procedures such as amniocentesis, cordocentesis, and chorionic villus sampling are associated with an increased risk of immunization to RhD. Recommendations are now in place for immune prophylaxis in the event of planned or unscheduled events that may increase FMH and therefore increase maternal exposure to RhD. For a listing of such events, see the Table. Anti-RhD prophylaxis policies vary slightly between developed countries (such as the United States, England, Mexico, Canada, and Australia). In third world countries, access to anti-RhD may still be extremely limited both owing to medical infrastructure reasons as well as socioeconomic reasons such as minimal prenatal care and delivery at home. In the United States, the Preventative Services Task Force current consensus recommendations are available at http://www.uspreventiveservicestaskforce.org (then search the site for Rh incompatibility). Current US recommendations include the proposal that all pregnant women should have antibody screening and RBC antigen typing for ABO and RhD at the rst prenatal visit. RhDnegative mothers with no antibody present at the rst
e86 NeoReviews Vol.14 No.2 February 2013

screen should have repeat screening at 24 to 28 weeks. If they remain negative, they should receive 1,500 IUs ( 300 mg) anti-RhD immunoglobulin at 28 weeks gestation unless the father of the infant is also known be RhD-negative. Second, women at high risk for FMH, such as previous transfusion, obstetrical complications in which anti-RhD was not administered, or the woman had sustained injuries, should continue to be screened for antibody development into the second and third trimesters. An additional dose of anti-RhD is recommended for women experiencing obstetrical complications with a risk for FMH or undergoing diagnostic testing that can increase FMH. This anti-RhD dose is trimester-dependent. For events in the rst trimester, the recommended dose is 250 IUs ( 50 mg). For events in the second or third trimester, the recommended dose is 1,500 IUs ( 300 mg). Last, the postnatal dose of 1,500 IUs is administered to all RhD-negative mothers. This dose should prevent immunization when as much as 15 mL of newborn blood enters the maternal circulation. Women experiencing highrisk problems or maneuvers, such as abruption, manual removal of the placenta, or multiple infant gestations may be at risk for higher volume FMH. An effort should be made to quantitate the FMH and administer additional antiRhD, if necessary (when estimated to be >15 mL).

Clinical Management: Identifying Pregnancies at Risk for Hemolytic Disease of the Fetus and Newborn
Monitoring is dependent upon previous pregnancy history, whether the infant from a previous pregnancy was clinically affected, which RBC antigen is involved, what is the fathers genotype and phenotype, how high is maternal antibody, and whether there is clinical evidence of fetal anemia. The rst level of monitoring is phlebotomy for serial determination of antiRBC antigen antibodies or indirect antiglobulin testing (IAT). An algorithm has been developed by Moise for monitoring of pregnancy in an RhD-negative woman with an RhD-positive fetus or RhD-positive father. This algorithm can also be applied to assessing risk for fetal anemia due to other RBC antigens. A rst pregnancy of an RhD-negative mother is monitored by repeated antiRBC antibody titers. If the titer remains low, serial monitoring continues and the infant is delivered at term. If the titer crosses above a critical threshold, then infants are monitored by serial fetal middle cerebral artery (MCA) velocities. An increased MCA velocity correlates well with fetal anemia. For pregnancies with previously affected infants, the current fetus is monitored with serial MCA velocities

disorders of blood

hemolytic disease

by using Doppler ultrasound to monitor for development of fetal anemia.

Clinical Management: Intrauterine


Once a pregnancy is identied as being at risk because of positive or rising IAT, serial Doppler ultrasound MCA velocities are utilized to monitor for fetal anemia. If mild anemia is detected, serial monitoring by ultrasound continues until there is adequate lung maturity or term delivery. Where severe anemia is suspected, cordocentesis may be utilized to conrm severe anemia (hematocrit <30% or hemoglobin <10 g/dL). In the event of severe anemia, an intrauterine transfusion may prevent progression to a severely ill, hydropic infant. Packed red blood cells (pRBCs) that are negative for the RBC antigen involved in the HDFN are used for transfusion. Additional pRBC specications include leukodepletion, cytomegalovirusnegative donor, and irradiated products to prevent transfusion-associated graft versus host disease.

3-hour period; 3 mL/kg pRBCs are needed to increase the hemoglobin 1 gm/dL (hematocrit 3%). Exchange transfusion was the mainstay of therapy early in management of RhD HDFN to minimize kernicterus from hyperbilirubinemic infants. Now, many hyperbilirubinemic infants who have HDFN will respond adequately to early phototherapy with blue light bilirubin lights, often in combination with bilirubin blankets. These infants may only require early phototherapy in combination with transfusion to address mild to moderate anemia. HDFN from some RBC antigens may result in delayed or prolonged postnatal anemia. Therefore a pediatric hematologist should be consulted to help monitor for the need for delayed or repeated pRBC transfusion.

Conclusions
RhD remains the most signicant antigen contributing to HDFN. However, prenatal anti-D prophylaxis has significantly decreased the incidence of severe HDFN. At this time, antibody prophylaxis is only available for HDFN due to RhD. Appropriate prenatal screening with IAT and Doppler ultrasound has greatly improved infant outcomes by allowing early identication of pregnancies at risk for HDFN. With appropriate early detection and multidisciplinary planning, these infants can be delivered in a timely manner with appropriate planning for postnatal resuscitation and postnatal therapy resulting in good neonatal outcomes.

Clinical Management: Postnatal


Delivery, either preterm or term, should be scheduled in a perinatal level 3 center with interdisciplinary obstetric, maternal-fetal, and pediatric services available. Assessment of fetal lung maturity and risk for subsequent respiratory distress must be taken into account in preparation for delivery. This includes consideration of glucocorticoids to accelerate lung maturity, when appropriate, and planning for personnel and equipment for resuscitation of the infant experiencing respiratory distress. A mildly or moderately anemic fetus will often tolerate labor adequately. A severely anemic fetus may not. ABO matched (if ABO typing has been done with previous cordocentesis) or O-type blood that is also negative for the antigen responsible for HDFN should be available to the resuscitating team in the delivery room. This pRBC product should be leukodepleted, cytomegalovirus-negative, and irradiated. The most severely ill infants may require an exchange transfusion, which involves replacing the native infant RBCs with appropriately antigen-negative RBCs to prevent further hemolysis. This is accomplished by replacing a total of 25 to 50 mL/kg pRBCs. Once vascular access has been established, 5 mL/kg aliquots can be removed and replaced over several minutes. This cycle is repeated until the targeted volume is replaced. Many anemic but more stable infants may be transfused with ABO-matched pRBCs that are negative for the antigen contributing to the HDFN at 10 mL/kg over a 2- to

American Board of Pediatrics Neonatal-Perinatal Content Specications


Know the diagnostic evaluation and perinatal management of fetal-maternal blood group incompatibility. Know the etiology and pathophysiology of hemolytic anemias in the neonate. Know the clinical and laboratory features of hemolytic anemia in the neonate. Know the management of hemolytic anemia in the neonate.

Suggested Reading
Basu S, Kaur R, Kaur G. Hemolytic disease of the fetus and newborn: Current trends and perspectives. Asian J Transfus Sci. 2011;5(1):37 Brinc D, Lazarus AH. Mechanisms of anti-D action in the prevention of hemolytic disease of the fetus and newborn. Hematology (Am Soc Hematol Educ Program). 2009: 185191
NeoReviews Vol.14 No.2 February 2013 e87

disorders of blood

hemolytic disease

Diamond LK, Blackfan KD, Baty JM. Erythroblastosis fetalis and its association with universal edema of the fetus, icterus gravis neonatorum and anemia of the newborn. J Pediatr. 1932;1(3):269309 Illanes S, Soothill P. Noninvasive approach for the management of hemolytic disease of the fetus. Expert Rev Hematol. 2009;2(5): 577582 Moise KJ Jr. Management of rhesus alloimmunization in pregnancy. Obstet Gynecol. 2008;112(1):164176 Osaro E, Charles AT. Rh isoimmunization in Sub-Saharan Africa indicates need for universal access to anti-RhD immunoglobulin

and effective management of D-negative pregnancies. Int J Womens Health (Larchmt). 2010;2:429437 Ross ME, Waldron PE, Cashore WJ, de Alarcon PA. Hemolytic disease of the fetus and newborn. In: de Alarcon PA, Werner EJ, Christensen RD, eds. Neonatal Hematology, Pathogenesis, Diagnosis, and Management of Hematologic Problems. 2nd ed. Cambridge, UK; Cambridge University Press; 2013 Stockman JA III. Overview of the state of the art of Rh disease: history, current clinical management, and recent progress. J Pediatr Hematol Oncol. 2001;23(6):385393

e88 NeoReviews Vol.14 No.2 February 2013

Hemolytic Disease of the Fetus and Newborn Mary Beth Ross and Pedro de Alarcn Neoreviews 2013;14;e83 DOI: 10.1542/neo.14-2-e83

Updated Information & Services References

including high resolution figures, can be found at: http://neoreviews.aappublications.org/content/14/2/e83 This article cites 6 articles, 0 of which you can access for free at: http://neoreviews.aappublications.org/content/14/2/e83#BIBL

Subspecialty Collections

This article, along with others on similar topics, appears in the following collection(s): Fetus and Newborn Infant http://neoreviews.aappublications.org/cgi/collection/fetus_newb orn_infant Information about reproducing this article in parts (figures, tables) or in its entirety can be found online at: /site/misc/Permissions.xhtml Information about ordering reprints can be found online: /site/misc/reprints.xhtml

Permissions & Licensing

Reprints

Index of Suspicion in the Nursery : Preterm Infant Born to Mother With High Fever Dinushan Kaluarachchi, Tomas Munoz, Suresh Khanna, Yekaterina Sitnitskaya and Benamanahalli Rajegowda Neoreviews 2013;14;e89 DOI: 10.1542/neo.14-2-e89

The online version of this article, along with updated information and services, is located on the World Wide Web at: http://neoreviews.aappublications.org/content/14/2/e89

Neoreviews is the official journal of the American Academy of Pediatrics. A monthly publication, it has been published continuously since . Neoreviews is owned, published, and trademarked by the American Academy of Pediatrics, 141 Northwest Point Boulevard, Elk Grove Village, Illinois, 60007. Copyright 2013 by the American Academy of Pediatrics. All rights reserved. Print ISSN: .

index of suspicion in the nursery

Preterm Infant Born to Mother With High Fever


Case Presentation
A 29-5/7-weeks-gestation female is delivered by urgent cesarean delivery and develops respiratory difculty soon after delivery. The mother is a 28-year-old primigravida who presented to the emergency department with high fever (102F), chills, and a sore throat. Results of a rapid inuenza antigen detection test (BinaxNOW Inuenza A & B, Alere Inc, Waltham, MA) on nasopharyngeal swab were positive for inuenza A and negative for inuenza B. Test results for respiratory syncytial virus and legionella were negative. Her clinical course was complicated by mutlilobar pneumonia and adult respiratory distress syndrome requiring intubation and mechanical ventilation. Results of repeat rapid inuenza antigen tests were negative at 36 hours after starting oseltamivir. The mother received one course of betamethasone 48 hours before the cesarean delivery (with membranes intact), which was performed because of worsening maternal condition. The infant weighed 1,500 g at birth and had Apgar scores of 9 at both 1 minute and 5 minutes. Because of respiratory distress, the infant underwent resuscitation (Neopuff, Fisher & Paykel Healthcare Limited, Panmure, Auckland, Australia) and was transferred to the NICU. She received one dose of surfactant and was immediately extubated to nasal continuous positive airway pressure. The infants initial chest radiograph showed mild haziness and a possible right upper lobe inltrate consistent with pneumonia. Results of the rapid inuenza antigen detection test (BinaxNOW Inuenza A & B) were positive for inuenza A on her nasopharyngeal secretions at 4 hours after birth. The infant was started on oseltamivir together with ampicillin and gentamicin. She received three doses of oseltamivir, and results of the repeat inuenza antigen tests on day 3 were negative for inuenza A. The infant clinically did well; her repeat chest radiograph performed on day 3 was unremarkable, and she was weaned off continuous positive airway pressure. Her complete blood cell count and electrolytes were within normal limits. Blood gases were also reported to be normal. Results of blood cultures were negative, and antibiotics were discontinued after 48 hours. The remainder of her neonatal course was uneventful. The mother had a rapid recovery after delivery. The mother had been offered u vaccine during her prenatal checkup, but she refused. Both maternal and infant inuenza immunoglobulin G levels were elevated; immunoglobulin M levels were lower than the normal reference range.

The reader is encouraged to write possible diagnoses for each case before turning to the discussion. We invite readers to contribute case presentations and discussions. Please inquire rst by contacting Dr. Philip at aphilip@stanford.edu.

Author Disclosure Drs Kaluarachchi, Munoz, Khanna, Sitnitskaya, and Rajegowda have disclosed no nancial relationships relevant to this article. This commentary does contain a discussion of an unapproved/investigative use of a commercial product/device.

Case Discussion
Pregnant women are a high-risk group for developing complications with inuenza A viral infection because of physiological changes that occur during pregnancy. Pregnant women are at greater risk of developing pneumonia, pulmonary edema, and other serious respiratory illness, as happened in our case. Pregnancy is understood to be associated with suppression of immunologic functions, with most cases of inuenza occurring during the second or third trimester and signicant numbers requiring intensive care. Mortality rates of inuenza among pregnant women range from approximately 7% to 10%; perinatal mortality is also high because there is a high rate of stillbirth and premature births. (1) The Centers for Disease Control and Prevention (CDC) recommends
NeoReviews Vol.14 No.2 February 2013 e89

index of suspicion in the nursery

immediate antiviral treatment for pregnant women who have inuenza, preferably within 48 hours of diagnosis. Early treatment is associated with lower risks of serious medical complications. (2) Later treatment is associated with higher maternal mortality. Oseltamivir is considered the drug of choice for inuenza infection and is recommended for pregnant women. Its use during pregnancy outweighs the risks. (2)(3) Except for the increased risk of late transient hypoglycemia, there are no increased risks of adverse birth outcomes. Women who have moderate to severe infection experience signicant improvement after delivery, as seen in our patient. Pregnant women and newborn infants are at greater risk of morbidity and mortality from inuenza infection. There are no adverse effects reported from the use of inuenza vaccine during pregnancy in either mothers or newborns. It has no detrimental effect on the growth and development of the fetus. (4) Studies have reported on the cost-effectiveness of maternal inuenza vaccination in reducing the rates and severity of diseases both in seasonal inuenza epidemics and pandemics. Despite these ndings, vaccine coverage remains low. The mother in our case refused inuenza vaccination. There is scant literature regarding transplacental passage of inuenza virus. Viremia is believed to occur infrequently, and thus vertical transmission seems to be uncommon. Picone et al (5) reported a possible case of transplacental passage of inuenza virus 4 hours after birth but the infant did have brief (few seconds) contact with the mother. In the last H5N1 pandemic, a woman who was infected with the same virus had the viral genome identied in placental cytotrophoblast and in fetal respiratory secretions.
e90 NeoReviews Vol.14 No.2 February 2013

In our patient, there was no contact between mother and infant. Inuenza A antigen was positive on infants nasopharyngeal secretions at 4 hours of age. Results of a repeat rapid antigen detection test were negative for inuenza A after three doses of oseltamivir. Results of placental cultures were negative, but no placental pathology was performed. Only inuenza A rapid antigen testing was done (specicity: 97%), but inuenza A polymerase chain reaction was not performed. This event seems to be a case of possible transplacental transmission of inuenza virus. Oseltamivir was approved during the April 2009 H1N1 pandemic to be used in infants younger than 1 year. The recommended dosage is 3 mg/kg per dose, twice a day. It is suggested that premature infants start with 1 mg/kg per dose, twice daily. (6) There are no data on optimal duration of treatment in premature infants. A retrospective study in 32 premature infants who received oseltamivir as prophylaxis or treatment reported that this medication was well tolerated except for mild transient adverse effects. (7) Another study reported a case of necrotizing enterocolitis during the treatment with oseltamivir, although a causal relationship could not be conrmed. In our case, no adverse effects were reported.

Yekaterina Sitnitskaya, MD, and Benamanahalli Rajegowda, MD, Division of Neonatology, Department of Pediatrics, Lincoln Hospital Medical and Mental Health Center, Bronx, NY)

American Board of Pediatrics Neonatal-Perinatal Content Specications


Know the epidemiology, clinical manifestations, diagnostic criteria, prevention, and management of perinatal infections with inuenza.

References
1. Pierce M, Kurinczuk JJ, Spark P,
Brocklehurst P, Knight M; UKOSS. Perinatal outcomes after maternal 2009/H1N1 infection: national cohort study. BMJ. 2011;342: d3214 2. Hiba V, Chowers M, Levi-Vinograd I, Rubinovitch B, Leibovici L, Paul M. Benet of early treatment with oseltamivir in hospitalized patients with documented 2009 inuenza A (H1N1): retrospective cohort study. J Antimicrob Chemother. 2011;66 (5):11501155 3. Donner B, Niranjan V, Hoffmann G. Safety of oseltamivir in pregnancy: a review of preclinical and clinical data. Drug Saf. 2010;33(8):631642 4. Blanchard-Rohner G, Siegrist CA. Vaccination during pregnancy to protect infants against inuenza: why and why not. Vaccine. 2011;29(43):75427550 5. Picone O, Bernabe-Dupont C, VauloupFellous C, et al. A suspected case of in utero transmission of inuenza A (H1N1) 2009 [in French]. J Gynecol Obstet Biol Reprod (Paris). 2011;40(5):473475 6. Acosta EP, Jester P, Gal P, et al; National Institute of Allergy and Infectious Diseases Collaborative Antiviral Study Group. Oseltamivir dosing for inuenza infection in premature neonates. J Infect Dis. 2010;202 (4):563566 7. Pannaraj PS, Tam B, Akan D. Oseltamivir treatment and prophylaxis in a neonatal intensive care unit during a 2009 H1N1 inuenza outbreak. J Perinatol. 2011;31(7):487493

Lessons for the Clinician


Vertical transmission of inuenza A is possible, and it is therefore important to vaccinate against inuenza during pregnancy. Planned early delivery in cases of severe maternal inuenza A infection may be benecial. Early treatment with neuraminidase inhibitors during pregnancy may minimize problems for the infant. Oseltamivir use in preterm infants seems to be safe. (Dinushan Kaluarachchi, MD, Tomas Munoz, MD, Suresh Khanna, MD,

Index of Suspicion in the Nursery : Preterm Infant Born to Mother With High Fever Dinushan Kaluarachchi, Tomas Munoz, Suresh Khanna, Yekaterina Sitnitskaya and Benamanahalli Rajegowda Neoreviews 2013;14;e89 DOI: 10.1542/neo.14-2-e89

Updated Information & Services References

including high resolution figures, can be found at: http://neoreviews.aappublications.org/content/14/2/e89 This article cites 7 articles, 3 of which you can access for free at: http://neoreviews.aappublications.org/content/14/2/e89#BIBL

Subspecialty Collections

This article, along with others on similar topics, appears in the following collection(s): Fetus and Newborn Infant http://neoreviews.aappublications.org/cgi/collection/fetus_newb orn_infant Information about reproducing this article in parts (figures, tables) or in its entirety can be found online at: /site/misc/Permissions.xhtml Information about ordering reprints can be found online: /site/misc/reprints.xhtml

Permissions & Licensing

Reprints

Strip of the Month: February 2013 Maurice L. Druzin and Nancy Peterson Neoreviews 2013;14;e91 DOI: 10.1542/neo.14-2-e91

The online version of this article, along with updated information and services, is located on the World Wide Web at: http://neoreviews.aappublications.org/content/14/2/e91

Neoreviews is the official journal of the American Academy of Pediatrics. A monthly publication, it has been published continuously since . Neoreviews is owned, published, and trademarked by the American Academy of Pediatrics, 141 Northwest Point Boulevard, Elk Grove Village, Illinois, 60007. Copyright 2013 by the American Academy of Pediatrics. All rights reserved. Print ISSN: .

strip of the month

Strip of the Month: February 2013


Maurice L. Druzin, MD,* Nancy Peterson, RNC, PNNP, MSN, IBLC

Electronic Fetal Monitoring Case Review Series


Electronic fetal monitoring (EFM) is a popular technology used to establish fetal well-being. Despite its widespread use, terminology used to describe patterns seen on the monitor has not been consistent until recently. In 1997, the National Institute of Child Health and Human Development (NICHD) Research Planning Workshop published guidelines for interpretation of fetal tracings. This publication was the culmination of 2 years of work by a panel of experts in the eld of fetal monitoring and was endorsed in 2005 by both the American College of Obstetricians and Gynecologists (ACOG) and the Association of Womens Health, Obstetric and Neonatal Nurses. In 2008, ACOG, NICHD, and the Society for Maternal-Fetal Medicine reviewed and updated the denitions for fetal heart rate (FHR) patterns, interpretation, and research recommendations. Following is a summary of the terminology denitions and assumptions found in the 2008 NICHD workshop report. Normal values for arterial umbilical cord gas values and indications of acidosis are dened in Table 1.

Author Disclosure Dr Druzin and Ms Peterson have disclosed no nancial relationships relevant to this article. This commentary does not contain a discussion of an unapproved/ investigative use of a commercial product/ device.

Assumptions From the NICHD Workshop


Denitions are developed for visual interpretation, assuming that both the FHR and uterine activity recordings are of adequate quality. Denitions apply to tracings generated by internal or external monitoring devices. Periodic patterns are differentiated based on waveform, abrupt or gradual (eg, late decelerations have a gradual onset and variable decelerations have an abrupt onset). Long- and short-term variability are evaluated visually as a unit. Gestational age of the fetus is considered when evaluating patterns. Components of FHR do not occur alone and generally evolve over time.

Denitions
Baseline FHR Approximate mean FHR rounded to increments of 5 beats per minute in a 10-minute segment of tracing, excluding accelerations and decelerations, periods of marked variability, and segments of baseline that differ by >25 beats per minute. In the 10-minute segment, the minimum baseline duration must be at least 2 minutes (not necessarily contiguous) or the baseline for that segment is indeterminate. Bradycardia is a baseline of <110 beats per minute; tachycardia is a baseline of >160 beats per minute. Sinusoidal baseline has a smooth sine wave-like undulating pattern, with waves having regular frequency and amplitude. Baseline Variability Fluctuations in the baseline FHR of 2 cycles per minute, uctuations are irregular in amplitude and frequency, uctuations are visually quantitated as the amplitude of the peak to trough in beats per minute. Classication of variability: Absent: Amplitude range is undetectable Minimal: Amplitude range is greater than undetectable to 5 beats per minute Moderate: Amplitude range is 6 to 25 beats per minute Marked: Amplitude range is >25 beats per minute
*Charles B. and Ann L. Johnson Professor of Obstetrics; Chief, Division of Maternal-Fetal Medicine; Co-Medical Director, MidCoastal California Perinatal Outreach Program, Stanford University School of Medicine, Palo Alto, CA. Director of Perinatal Outreach, Stanford University, Palo Alto, CA.

NeoReviews Vol.14 No.2 February 2013 e91

strip of the month

Table 1.

Arterial Umbilical Cord Gas Values


pH PCO2 (mmHg) <60 (35 to 70) >60 <60 >60 PO2 (mmHg) 20 Variable Variable Variable Base Excess L10 (L2.0 to L9.0) L10 L10 L10
a

Normal Respiratory acidosis Metabolic acidosis Mixed acidosis


a

7.20 (7.15 to 7.38) <7.20 <7.20 <7.20

Normal ranges from Obstet Gynecol Clin North Am. 1999;26:695.

Accelerations Abrupt increase in FHR above the most recently determined baseline. Onset to peak of acceleration is <30 seconds, acme is 15 beats per minute above the most recently determined baseline and lasts 15 seconds but <2 minutes. Before 32 weeks gestation, accelerations are dened by an acme of 10 beats per minute above the most recently determined baseline for 10 seconds. Prolonged acceleration lasts 2 minutes but <10 minutes. Late Decelerations Gradual decrease in FHR (onset to nadir 30 seconds) below the most recently determined baseline, with nadir occurring after the peak of uterine contractions. Considered a periodic pattern because it occurs with uterine contractions. Early Decelerations Gradual decrease in FHR (onset to nadir 30 seconds) below the most recently determined baseline, with nadir occurring coincident with uterine contraction. Also considered a periodic pattern. Variable Decelerations Abrupt decrease in FHR (onset to nadir <30 seconds). Decrease is 15 beats per minute below the most recently determined baseline lasting 15 seconds but <2 minutes. May be episodic (occurs without a contraction) or periodic. Prolonged Decelerations Decrease in the FHR 15 beats per minute below the most recently determined baseline lasting 2 minutes but <10 minutes from onset to return to baseline. Decelerations are tentatively called recurrent if they occur with 50% of uterine contractions in a 20-minute period.
e92 NeoReviews Vol.14 No.2 February 2013

Decelerations occurring with <50% of uterine contractions in a 20-minute segment are intermittent. Sinusoidal FHR Pattern Visually apparent, smooth sine wave-like undulating pattern in the baseline with a cycle frequency of 3 to 5 per minute that persists for 20 minutes. Uterine Contractions Quantied as the number of contractions in a 10-minute window, averaged over 30 minutes. Normal: 5 contractions in 10 minutes Tachysystole: >5 contractions in 10 minutes

Interpretation
A three-tier FHR interpretation system has been recommended as follows: Category I FHR tracings: Normal, strongly predictive of normal fetal acid-base status and require routine care. These tracings include all of the following: Baseline rate: 110 to 160 beats per minute Baseline FHR variability: Moderate Late or variable decelerations: Absent Early decelerations: Present or absent Accelerations: Present or absent Category II FHR tracings: Indeterminate, require evaluation and continued surveillance and reevaluation. Examples of these tracings include any of the following: Bradycardia not accompanied by absent variability Tachycardia Minimal or marked baseline variability Absent variability without recurrent decelerations Absence of induced accelerations after fetal stimulation Recurrent variable decelerations with minimal or moderate variability Prolonged decelerations Recurrent late decelerations with moderate variability

strip of the month

Variable decelerations with other characteristics, such as slow return to baseline Category III FHR tracings: Abnormal, predictive of abnormal fetal acid-base status and require prompt intervention. These tracings include: Absent variability with any of the following: Recurrent late decelerations Recurrent variable decelerations Bradycardia Sinusoidal pattern Data from Macones GA, Hankins GDV, Spong CY, Hauth J, Moore T. The 2008 National Institute of Child Health and Human Development workshop report on electronic fetal monitoring. Obstet Gynecocol. 2008;112: 661666 and American College of Obstetricians and Gynecologists. Intrapartum fetal heart rate monitoring: nomenclature, interpretation, and general management principles. ACOG Practice Bulletin No. 106. Washington, DC: American College of Obstetricians and Gynecologists; 2009. We encourage readers to examine each strip in the case presentation and make a personal interpretation of the

ndings before advancing to the expert interpretation provided.

Case Presentation
History
Patient is a 19-year-old G1P0 Hispanic woman. She had an uncomplicated prenatal course. Her early prenatal screening testing, rst and second trimesters, was entirely within normal limits. At approximately 35 weeks gestation, she developed itching of the palms and soles of the feet that worsen at night. Serum bile acid levels were evaluated, and the results were 40 mmol/L consistent with moderate/severe intrahepatic cholestasis of pregnancy (IHCP).

Case Progression
Induction of labor was scheduled for 37 weeks gestation. On admission, a cervical examination showed that the cervix was 1 cm dilated, vertex at 3 station. An FHR admission tracing is obtained shortly after admission to labor and delivery (Fig 1).

Figure 1. EFM Strip #1.

NeoReviews Vol.14 No.2 February 2013 e93

strip of the month

Figure 1. EFM Strip #1.

Findings from EFM Strip #1 are: Variability: Moderate Baseline rate: 130 beats per minute Episodic patterns: Accelerations Periodic patterns: None Uterine contractions: Difcult to determine, monitor needs to be readjusted, but contractions appear to be 1 to 6 minutes long. Palpate for intensity and tone Interpretation: Category I tracing Differential diagnosis: Normal FHR tracing Action: Elevated levels of serum bile acids (>40 mmol/L) have been linked to adverse fetal outcomes, including increased risk of preterm delivery, meconium staining of the amniotic uid, fetal bradycardia,

fetal distress, and fetal demise. Most experts recommend delivery by 37 weeks gestation to decrease the risk of these complications to the fetus. At this time, the fetus is well oxygenated as evidenced by accelerations and moderate variability. An induction was started with Pitocin (oxytocin), and, over the next 8 hours, the physician ruptured the membranes, obtaining clear uid, and the patient received an epidural. The FHR remained in the 130 to 140 range with moderate variability with accelerations and no decelerations. The cervix was noted to be 3 cm, 90% effaced, and 2 station. Five hours later, the cervix was 7 cm, 90% effaced, and 1 station. The following tracing is obtained 50 minutes later (Fig 2).

Figure 2. EFM Strip #2.


e94 NeoReviews Vol.14 No.2 February 2013

strip of the month

Figure 2. EFM Strip #2.

Findings from EFM Strip #2 are: Variability: Moderate Baseline rate: 135 beats per minute Episodic patterns: None Periodic patterns: Intermittent variable decelerations Uterine contractions: Tachysystole pattern with contractions every 1 to 1.5 minutes, palpate for intensity and tone Interpretation: Category II Differential diagnoses: Indeterminate FHR tracing with evidence of cord compression

Action: The moderate variability is reassuring and is reective of a well-oxygenated fetus. Interventions such as repositioning the mother to a lateral position, and intravenous hydration can generally resolve the cord compression pattern. In addition to these interventions, efforts should be made to reduce uterine activity by decreasing the Pitocin (oxytocin) infusion to allow for improved fetal oxygenation. Ten minutes later, an examination reveals that the cervix is completely dilated and 100% effaced and 1 station. The patient is instructed to begin pushing and, 10 minutes later, the following tracing is obtained (Fig 3).

Figure 3. EFM Strip #3.


NeoReviews Vol.14 No.2 February 2013 e95

strip of the month

Figure 3. EFM Strip #3.

Findings from EFM Strip #3 are: Variability: Moderate Baseline rate: 140 beats per minute increasing to 150 toward the end of the tracing Episodic patterns: None Periodic patterns: Recurrent variable decelerations Uterine contractions: Unable to determine because of pushing and the inability to pick up contractions. Monitor needs to be readjusted. Palpate for tone, frequency, and intensity. Interpretation: Category II Differential diagnosis: Same Action: Variable decelerations are caused by cord compression and can usually be resolved with maternal

lateral positioning, and intravenous hydration. However, when they are recurrent and do not improve despite the interventions, further evaluation is warranted. An amnioinfusion (installation of uid into the amniotic cavity via an intrauterine catheter) can be very effective at eliminating recurrent variable decelerations. In addition, the physician should be notied to come in and evaluate the patient. Over the next hour, the patient continues to push with worsening recurrent variable deceleration down to 60 beats per minute with slow return to an increasing baseline 160 to 170 beats per minute. The fetal tracing is shown in Fig 4.

Figure 4. EFM Strip #4.


e96 NeoReviews Vol.14 No.2 February 2013

strip of the month

Figure 4. EFM Strip #4.

Findings from EFM Strip #4 are: Variability: Minimal to moderate Baseline rate: 165 beats per minute Episodic patterns: Prolonged deceleration Periodic patterns: Difcult to determine because of the inability to pick up contractions, but there appear to be recurrent variables and late appearing decelerations Uterine contractions: Unable to determine. Monitor needs to be readjusted to pick up the contractions. Interpretation: Category II Differential diagnosis: Cord compression

Action: Interventions should continue to focus on improving blood ow to the placenta and fetus (lateral position, hydration, and discontinuing the Pitocin [oxytocin]). The presence of variability is reassuring and reective of an intact central nervous system and adequate oxygen pathway. Over the next 45 minutes, the FHR decelerations continue to worsen with decreased variability. A decision is made to attempt an operative vacuum-assisted vaginal delivery. Request the presence of the NICU team at the delivery and notify the anesthesiologist, as well, to be available if the vacuum-assisted delivery is not successful. The nal tracing is shown in Fig 5.

Figure 5. EFM Strip #5.


NeoReviews Vol.14 No.2 February 2013 e97

strip of the month

Figure 5. EFM Strip #5.

Findings from EFM Strip #5 are: Variability: Minimal Baseline rate: Unable to determine Episodic patterns: Indeterminate Periodic patterns: Indeterminate because of inadequate uterine contraction tracing Uterine contractions: Readjust the monitor and palpate contractions Interpretation: Category II evolving to category III Action: Prepare to expedite the delivery.

severe and intolerable. It is predominantly present on the palms and soles of the feet, but may often be generalized. The itching often becomes worse at night. The clinical presentation of pruritus, combined with elevated bile acids, in the absence of any other illness that may produce similar symptoms, conrms the diagnosis of IHCP. Liver enzymes are commonly elevated, and, in those cases, other common pregnancy complications such as preeclampsia and HELLP syndrome (Hemolysis, Elevated Liver enzymes, Low Platelet count) must be ruled out. The disease is divided into three categories based on the level of bile acids (2): Mild IHCP: 10 to 19 mmol/L Moderate IHCP: 20 to 39 mmol/L Severe IHCP: > 40 mmol/L In general, there are few maternal risks to this disease, because it resolves after delivery. Birth control pills containing estrogen may cause an exacerbation of this disease in nonpregnant women. IHCP carries signicant risk for the fetus. The major complications are prematurity, meconium-stained amniotic uid, intrauterine demise, and an increased risk for neonatal respiratory distress syndrome.

Outcome
After three pulls with the vacuum, a successful vaginal delivery is accomplished with a 2,600-g male infant. Apgar scores are 4 and 8 at 1 and 5 minutes, respectively. The arterial cord gases noted in Table 2 are reective of a mild mixed metabolic acidosis.

Discussion
IHCP occurs in the second and third trimesters and is characterized by intense pruritus and an elevation in serum bile acid concentration. (1) The pruritus is often

Table 2.

Arterial Umbilical Cord Gas Results


pH PCO2 (mmHg) <60 >60 <60 >60 86.4 PO2 (mmHg) 20 Variable Variable Variable 13 Base Excess L10 L10 L10 L10 L11
a

Normal Respiratory acidosis Metabolic acidosis Mixed acidosis Patient


a

7.20 <7.20 <7.20 <.20 7.0

Normal ranges from Obstet Gynecol Clin North Am. 1999;26:695.

e98 NeoReviews Vol.14 No.2 February 2013

strip of the month

Treatment consists of the use of ursodiol, which will often decrease the level of bile acids and ameliorate the pruritus. The increased risk of adverse fetal outcomes including intrauterine demise occurs mainly after 37 weeks gestation. The current accepted approach to IHCP is delivery at 37 to 38 weeks, (3) even though the data are limited. Weekly antepartum testing at the onset of the disease is recommended before 37 weeks. However, there are isolated reports of fetal death despite reassuring antepartum fetal testing. The exact mechanism causing fetal complications is still unknown, but it may be related to level of bile acids (40 or greater) and some undened placental toxicity. Vasospasm of the placental chorionic surface vessels and/or sudden development of fetal

arrhythmia have been postulated to be possible mechanisms of fetal demise.

References
1. Bacq Y, Lee RH. Up-to-Date: Intrahepatic cholestasis of
pregnancy. Literature review current through October 2012; Topic last updated on November 1, 2012. Available at : http://www. uptodate.com/contents/intrahepatic-cholestasis-of-pregnancy. Accessed January 8, 2013 2. Lee RH, Kwok KM, Ingles S, et al. Pregnancy outcomes during an era of aggressive management for intrahepatic cholestasis of pregnancy. Am J Perinatol. 2008;25(6):341345 3. Spong CY, Mercer BM, Dalton M, Kilpatrick S, Blackwell S, Saade G. Timing of indicated late-preterm and early-term birth. Obstet Gynecol. 2011;118(2 pt 1):323333

Answer Key for February Issue: Neonatal Cholestasis: 1. B; 2. D; 3. D; 4. D; 5. C Neonatal Thrombocytopenia: 1. E; 2. C; 3. C; 4. C; 5. D


NeoReviews Vol.14 No.2 February 2013 e99

Strip of the Month: February 2013 Maurice L. Druzin and Nancy Peterson Neoreviews 2013;14;e91 DOI: 10.1542/neo.14-2-e91

Updated Information & Services References

including high resolution figures, can be found at: http://neoreviews.aappublications.org/content/14/2/e91 This article cites 2 articles, 0 of which you can access for free at: http://neoreviews.aappublications.org/content/14/2/e91#BIBL

Subspecialty Collections

This article, along with others on similar topics, appears in the following collection(s): Fetus and Newborn Infant http://neoreviews.aappublications.org/cgi/collection/fetus_newb orn_infant Information about reproducing this article in parts (figures, tables) or in its entirety can be found online at: /site/misc/Permissions.xhtml Information about ordering reprints can be found online: /site/misc/reprints.xhtml

Permissions & Licensing

Reprints

Das könnte Ihnen auch gefallen